You are on page 1of 85

The PCN PEP STUDY BUDDY is specially designed and formulated to aid intending PEP candidates study

better and smarter. The package contains an abundance of questions sourced from past PEPs and other
reliable educational sources. All aspects of the exam are covered here, and are categorized as follows:
1. Clinical Pharmacy/Pharmaceutical Care.
2. Pharmacoeconomics, Pharmacy Management and General Management.
3. Pharmacology and Pharmacotherapy.
4. Pharmaceutical Technology and Dispensing.
5. Public Health, and Infectious Diseases.
6. Pharmacy Laws, Administration, Jurisprudence, and Ethics.
7. Pharmacognosy.
8. Veterinary Pharmacy.

CLINICAL PHARMACY/PHARMACEUTICAL CARE

1. Which of the following holistically captures pharmacists’ role in health care?


(a) Distribution (b) Clinical Pharmacy (c) Pharmaceutical Care (d) Total Pharmacy care

2. How many components are there in pharmaceutical care philosophy as espoused by Hepler
and Strand?
(a) Four (b) Three (c) Two (d) One

3. Pharmacists’ responsibility in pharmaceutical care is to:


(a) Counsel patients (b) Dispense medications (c) Identify common symptoms (d) Resolve drug
therapy problems (DTPs)

4. The most beneficial outcome of drug therapy to the patient is:


(a) Clinical (b) Economic (c) Humanistic (d) Process

5. “Length of stay” in a care facility is an example of which outcome?


(a) Clinical (b) Economic (c) Humanistic (d) Process

1
6. Which of the following outcomes in malaria therapy may result in resistance development?
(a) Clinical cure (b) Parasitological Cure (c) Radical Cure (d) None of the options
7. The “cornerstone” step in the provision of pharmaceutical care is to:
(a) Establish professional relationship (b) Collect patient data (c)
Identify drug therapy problems (d) Document activities

8. Data collected through patient interview is often described as:


(a) Inaccurate (b) Accurate (c) Subjective (d) Objective

9. One of the following is regarded as a novel pharmaceutical care behavior:


(a) Monitor patient outcomes (b) Obtain patient symptom information
(c) Make a recommendation (d) Obtain patient medication history

10. Which of the following components of quality is the most important to the patient?
(a) Structure (b) Process (c) Outcome (d) None of the above

11. Effectiveness of medicinal products is determined in:


(a) Laboratory evaluation (b) Clinical trial (c) Real world (d) Post-marketing surveillance

12. Which of the following is not correct regarding drug therapy problems?
(a) Medical problem (b) Negative outcome (c) Amenable to detection (d) None of the options

13. Drug related needs of patients include the following EXCEPT:


(a) Appropriate indication (b) Effectiveness (c) Safety (d) Treated indication

14. An asthmatic patient was given propranolol for palpitations. The potential DTPs are:
(a) Unnecessary drug therapy and untreated indication
(b) Wrong drug and adverse drug reaction
(c) Inappropriate adherence and dosage too high
(d) Wrong drug and non-adherence
15. Pharmacists communicating with patient should NOT:
(a) Appear relaxed (b) Use gestures (c) Use 100% eye contact (d) Be positioned at patients’ eye level

2
16. “How do you take your antihypertensive medication?” Is an example of:
(a) Open ended question (b) Close ended question (c) Leading question (d) Direct question

17. Pharmacist-patient communication should avoid this question type:


(a) Open ended (b) Close ended (c) Leading (d) Probing

18. The procedure used to evaluate structures lying 4-5 cm under the skin is called:
(a) Inspection (b) Palpation (c) Percussion (d) Auscultation

19. Beta-blocker induced bradycardia is physically assessed using:


(a) Inspection and Palpation (b) Auscultation and Percussion (c) Palpation and
Auscultation (d) Percussion and Palpation

20. One of the following is a direct method of assessing adherence:


(a) Pill count (b) Urine assay (c) Refill record (d) Measurement of health outcome

21. Patient adherence is an example of intermediate:


(a) Clinical outcome (b) Humanistic outcome (c) Economic outcome (d) None of the
options

22. Strategies to improve adherence include the following EXCEPT:


(a) Promote self-efficacy (b) Empower patients (c) Fear tactics (d) Telephone call

23. The first stage in developing collaboration between pharmacists and physicians is:
(a) Professional awareness (b) Professional recognition
(c) Exploration and trial (d) Commitment to collaboration

24. Characteristics of the pharmaceutical care product include the following except:
(a) Inconsistent (b) Intangible (c) Inventoried (d) Inseparable

25. When marketing pharmaceutical care, which of the following Ps is in addition to those
encountered in the marketing of pharmaceuticals?
(a) Promotion (b) Position (c) Product (d) Place

3
26. Factors necessitating individualized pharmaceutical care in the elderly include the following
EXCEPT:
(a) Multiple diseases (b) Polypharmacy (c) Adherence (d) Physiological changes

27. Off-label medication use is:


(a) Applicable to children only (b) Unlicensed indication (c) Medication use with marketing
authorization
(d) All of the options

28. Patient-focused intervention in hypertension includes the following EXCEPT:


(a) Education (b) Adherence promotion (c) Dosage adjustment (d) Low salt diet

29. Which of these interventions is beneficial in congestive heart failure?


(a) Moderate smoking (b) Moderate alcohol (c) Moderate exercise (d) All of the options

30. Dyspnea experienced in heart failure is due to:


(a) Left ventricular hypertrophy (b) Right ventricular hypertrophy (c) Pulmonary edema
(d) Peripheral edema

31. Long term glycemic control is monitored with:


(a) Glycosylated hemoglobin (b) Blood pressure (c) Fasting blood glucose (d) Fasting lipid profile

32. Concerning inhalers, which of the following statements is CORRECT?


(a) In children, breath-actuated inhalers are preferred to metered-dose inhaler with a
spacer.
(b) Improper respiratory function has no effect on the use of breath actuated inhalers.
(c) The disadvantage of breath-actuated inhalers is that there is need to coordinate
actuation with inhalation.
(d) The disadvantage of metered-dose inhalers can be overcome through the use of a
spacer.

33. Pharmacists who encourage patients with diabetes or heart disease to take their medications
appropriately are involved in:
4
(a) Primary prevention (b) Secondary prevention (c) Specialist prevention (d) Tertiary prevention
34. An eye drop, once opened, should not be used beyond
(a) 1 week (b) 2 weeks (c) 4 weeks (d) Expiration date

35. Which of the following contraceptive methods can predispose women to sexually
transmitted infections?
(a) Oral pills (b) Intrauterine devices (c) Injectable contraceptives (d) Implants

36. Which of the following antihypertensive drugs would also benefit a patient with benign
prostatic hyperplasia?
(a) Telmisartan (b) Sotalol (c) Doxazosin (d) Clonidine

37. By convention, 24G IV cannular is identified by which color?


(a) Red (b) Yellow (c) Green (d) Blue

38. Use of low dose aspirin prevents:


(a) Hypertensive crisis (b) Stroke (c) Arrhythmia (d) Cardiomyopathy

39. Which of these would confer passive immunization?


a) Administration of live attenuated organism
b) Administration of antigenic portion of organism
c) Administration of immunoglobulin against the causative organism
d) Administration of dead organism

40. The urine M/C/S for Mrs. Ada reads as follows:


Ciprofloxacin 3+, Gentamycin 1+, Cefixime 2+, Nalidixic acid 2+
Given that Mrs. Ada has a previous history of reaction to moxifloxacin, which of the drugs
above would you recommend for her treatment?
(a) Ciprofloxacin (b) Gentamycin (c) Cefixime (d) Nalidixic Acid

41. The traditional drug dispensing involves the following EXECEPT

5
(a) Responsibility for drug treatment (b) Dispensing medicines (c) Drug related preparation
emergence (d) Patient education and counseling.
42. Which of the following does not form a core part of pharmaceutical care?
(a) health promotion (b) medication reconciliation (c) advice to patients (d) diagnosis of medical
problems

43. The four basic components of pharmaceutical care philosophy as espoused by helper and
strand include the following EXCEPT:
(a) caring (b) pharmacist responsibilities (c) social need (d) standards and guidelines

44. The process of optimizing beneficial outcomes and reducing harm from drugs including
appropriateness, monitoring and advice to patients pertains to:
(a) medication review (b) drug related problems (c) medicines management (d) drug dispensing

45. In establishing and maintain a professional relation, interaction between the pharmacist and
the patient must occur to assure a relationship based on all these EXCEPT:
(a) transparent communication (b) caring (c) patient’s active decision (d) open cooperation.

46. The goal of pharmaceutical care can be described by the attainment of any of these EXCEPT:
(a) optimal health related quality of life (b) improved clinical outcomes (c) measurable cost of
care (d) increment of target symptoms.

47. Which of the following does not describe a drug therapy problem, it could:
(a) be an undesirable event (b) be a potential or actual problem (c) be a medical problem (d)
interfere with desired outcome.

48. A drug therapy problem arising from not taking a drug correctly can be regarded as:
(a) unnecessary drug therapy (b) inappropriate adherence (c) drug interaction (d) untreated
indication.

49. The following words are used to describe pharmaceutical care EXCEPT:
(a) event (b) process (c) collaboration (d) outcome.

6
50. Which of the following outcomes is not public health oriented?
(a) cure of disease (b) prevention of drug induced disease (c) illness prevention (d) none of the
options.
51. Guidelines to patient counseling include which one of the following
a. Ascertain the right patient
b. Collect data on medical history
c. Determine prescriber’s intention before intervention
d. All of the above

52. The following is regarded as corner-stone to provision of pharmaceutical care:


(a) professional relation (b) subjective and objective data (c) identifying drug therapy problem
(d) documentation.

53. Which of the following pharmaceutical care plans is not properly written? (a) normalize
blood pressure (b) maintain blood pressure below 140/90 mmHg (c) maintain pre-analysis (d)
cost-minimization analysis.

54. The patient that has a medical condition that requires the initiation of new therapy is said to
have; (a) unnecessary drug therapy (b) additional drug therapy problem (c) wrong drug therapy
(d) inappropriate adherence.

55. An attempt to introduce pharmaceutical care should develop a minimum of: (a) a
practitioner and a documentation system (b) practitioner and information system (c) patients
and data base (d) all of the options.

56. Pharmaceutical care can be reduced to: (a) Critical thinking and problem solving (b) drug
therapy problems and identification (c) structures and processes (d) managing therapy and
communication.

57. The test and treat approach to malaria therapy is to ensure: (a) clinical cure (b) parasitological
cure (c) radical cure (d) all of the options
7
58. The most important barrier to pharmaceutical care is: (a) lack of access to patient records (b)
time (c) pharmacist’s attitude (d) lack of enabling law

59. Pharmaceutical care does not depend on:


(a) setting (b) practitioner (c) patients (d) documentation

60. Which of the following diseases does not have target symptoms? (a) diabetes (b) asthma (c)
hypertension (d) malaria

61. In which of the following diseases can we not describe the clinical outcome as cure? (a)
malaria (b) syphilis (c) tuberculosis (d) acquired immune deficiency syndrome

62. Patient satisfaction with pharmaceutical service is a measure of: (a) clinical outcome (b)
economical outcome (c) humanistic outcome (d) all of the options.

63. The difference between pharmaceutical care (pc) and clinical pharmacy (cp) based patient
care is that:
a) CP is cyclical in nature, while pc is episodic
b) PC is patient-oriented pharmacy care, while CP is not patient-oriented.
c) PC is cyclical in nature while CP is episodic
d) The difference between PC and CP is a matter of semantics

64. Drug Therapy problems (DTPs) are divided into:


(a) Actual and patient-focused DTPs
(b) Potential and drug-focus DTPs
(c) Actual and potential DTPs
(d) Actual potential and “I need more information” DTPs

65. During patient counseling, the pharmacist found out that a patient has had a history of
immediate hypersensitivity reaction to amoxicillin, such a drug therapy problem is classified as:
(a) Wrong drug – actual DTP
(b) Adverse drug reaction – actual DTP
(c) Needs additional drug therapy – actual DTP

8
(d) Wrong drug and adverse drug reaction

66. Focusing PC service on patients with a particular disease typically requires developing the
following except:
(a) Specific care protocols
(b) Patient education materials
(c) Provider communication tools
(d) Quality assurance protocol for medicines

67. Subjective data are often supplied by the patient such as all but one of the following: (a)
Medical (b) chief complaint (c) history of present illness (d) vital sign
68. One of the following is not a vital sign: (a) Body temperature (b) Blood pressure (c) body
weight (d) respiratory rate.

69. Pharmaceutical care is essentially:(a) A practice ideal (b) development of documentation (c)
Computerization of pharmacy (d) Pharmacokinetic analysis of drugs in plasma.

70. The “cornerstone” in pharmaceutical care is: (a) development of rapport with patient (b)
Patient data collection (c) Patient data evaluation (d) development of professional relationship.

71. Two basic requirements for pharmaceutical care are:


(a) Infrastructure and legal frame work
(b) Practitioner and documentation system
(c) Legal framework and documentation system
(d) Practitioner and legal framework

72. The pharmacist’s responsibility in pharmacy care is to:


(a) Collect patient specific data
(b) Evaluate patient data
(c) Resolve drug therapy problems
(d) All of the options

73. Why should patients be instructed to rinse well after using a corticosteroid inhaler?
a. To minimize headaches
b. To prevent candidiasis

9
c. To increase therapeutic effects
d. All of the options

74. Which of the following dosage forms would not be recommended for a patient who has an
allergy to alcohol?
a. Slow-release tablets b. Elixirs c. Pastilles d. Syrups
75. Which of the following drugs should be taken on empty stomach?
a. Naproxen b. Levothyroxine c. Prednisolone d. Nitrofurantoin
76. Which auxiliary label is most appropriate for the prescription of Amoxicillin 250mg/mL
suspension?
a. Shake the bottle before use
b. Take with food

c. Avoid exposure to sunlight


d. All of the options

77. A patient complained of brown staining teeth when he uses a particular mouthwash. Which
of the following could be the culprit?
a. Thymol b. Chlorhexidine c. hydrogen peroxide d. All of the options

78. With respect to adverse drug reactions, the science of pharmacovigilance includes all the
following except:
a. Detection b. Assessment c. Prevention d. Management

79. An untoward medical occurrence in a patient on a medical product which does not
necessarily have to have a causal relationship with the treatment is called:
a. Adverse drug event
b. Adverse drug reaction

c. Side effect of a drug

d. Hypersensitivity reaction

80. Which of the following is not an iatrogenic disease?


a. Peptic ulcer while on corticosteroid

10
b. Hepatitis while on isoniazid

c. Parkinsonism while on antipsychotics


d. Crohn’s disease while on paracetamol

81. A serious adverse event/reaction is any untoward medical occurrence that is/does all of the
following except:
a. Occurs at low doses
b. Is life threatening
c. Requires or extends patient hospitalization
d. Results in death

82. With respect to the onset of an adverse event following drug administration:
a. An acute ADR is one that occur within the first 15 minutes
b. A sub-acute ADR is one that occurs within the first 24 hours
c. A sub latent ADR is one that occurs within a few days
d. All of the options

83. Which of the following is not good advice for slowing cataract development
a. Avoiding or stopping smoking

b. Avoiding the use of mydriatics


c. Avoiding tanning booths
d. Avoiding the use of steroids

84. Gluten-free diet is recommended for people with


a. Inflammatory bowel disease
b. Gastroenteritis
c. Crohn’s disease
d. Coeliac disease

85. Type 2 diabetes may be indicated by what type of bad breath?

11
a. Cheesy b. Fruity c. Fishy d. Acidic

86. A diabetic patient who is also hypertensive may benefit most from which of the following
antihypertensive drug classes?

a. ARBs b. ACEIs c. CCBs d. BBs

87. Those caring for asthmatic school children should always carry the following medication:

a. Corticosteroid inhaler

b. Corticosteroid tablets

c. Salbutamol inhaler

d. Salbutamol tablets

88. Which of the following results can be considered a normal blood pressure reading for a
woman over 60 years of age?

a. 110/79mmHg

b. 134/87mmHg

c. 148/93mmHg

d. 150/98mmHg

89. Patient AY complained of nausea, vomiting, abdominal cramps, and bloody watery stools. AY
is likely to have?

a. Drug induced acute diarrhea

b. Bacterial gastroenteritis

c. Protozoal gastroenteritis

d. Viral enterocolitis

90. In the education and management of an asthmatic patient, the following are recommended
except?
12
a. The patient should have an understanding of the action of each of the prescribed medicines.
b. The choice of the inhalation device(s) should be appropriate.
c. The patient should be educated on proper mode of usage of drugs and inhalation devices.
d. A general management plan should suffice for all patients.

91. A spacer device is best used:


a. With young children who would have difficulty coordinating proper MDI techniques.
b. With a DPI diskus to improve particle delivery into the lower lungs.
c. To increase the size of particles expelled from an inhaler and improve drug delivery.
d. To improve patient adherence to inhaled medications.
92. The doctor in the out-patient department placed a patient on a drug that Sanofi-Aventis
markets as Robaxin®. The generic name of the drug is methocarbamol. What is the drug indicated
for? a. Fibromyalgia b. Muscle spasms c. Insomnia d. Anxiety

93. While on call, you were asked by a physician to suggest a dopamine agonist available at the
pharmacy. Assuming you have all the under-listed drugs available, which would you suggest?
a. Risperidone b. Tizanidine c. Apomorphine d. Dipyridamole

94. Mrs. Mercy John is a regular client at your pharmacy. Today, she’s complaining about a tissue
nematode infection and asks you what kind of medicine she can take. Which of the following
would you not recommend?
a. Albendazole b. Metronidazole c. Diethylcarbamazine d. Praziquantel

95. A patient and her 6-yr old son walk into your pharmacy with a prescription:
Syr. Ethosuximide (250mg/5ml)
10ml od x 1/52,
Then 15ml od x 1/52,
Then 20ml od x 1/52,
What is her son being treated for?
a. ADHD b. Epilepsy c. Nocturnal enuresis d. Neuralgia

96. During patient counselling, you told Mr. Audu Bala that the drug prescribed for him may
cause some of the following common side effects: nausea and vomiting, excessive growth of
13
body hair and gum tissue, ataxia, discolored urine, and sedation. Which of the following drugs
was prescribed for Mr. Bala?
a. Diazepam b. Amoxicillin c. Allopurinol d. Phenytoin

97. Which one of the following rules gives an error when used to calculate dosage for children
over 1 year of age?
a. Young’s rule
b. Clarke’s rule
c. Fried’s rule
d. Body surface area
98. You receive a prescription: Nitrofurantoin 50mg 1 tab po qid x 4/7. Which of these is most
likely the condition being treated?
a. Urinary tract infection
b. Hypertension
c. Candida infection
d. Acid reflux

99. The following may be considered a critical inhaler error except?


a. Exhaling into the device
b. Holding inhaler upside down
c. Holding inhaler upright
d. Failure to synchronize inhalation with device actuation

100. The most important factor to consider in patient counselling is?


a. Pharmaceutical care
b. Pharmacist’s knowledge
c. Documentation
d. Confidentiality

101. Adding life to years and adding years to life means?


a. Improving quantity and quality of life
b. Improving quality and quantity of life
c. Improving life expectancy and median age
14
d. Improving median age and life expectancy

102. One of the following is not correct regarding pharmaceutical care outcomes
a. Outcomes should be determined after therapy.
b. Outcomes should be pre-determined before therapy.
c. Outcomes should be specific to patient.
d. Outcomes can be economic, clinical, or humanistic.

103. What is hemoglobin A1c goal in patients who have diabetes?


a. <7% b. <8% c. <9% d. <10%
104. Pharmaceutical care goals in diabetes indicate:
a. Glycemic control
b. Blood pressure control
c. Cholesterol control
d. All of the above

105. A 35-year old woman presents with red and itchy eczematoid dermatitis. She had a dental
procedure earlier in the day, and the dentist administered a local anesthetic. There were no
other findings, although she indicated that she had a history of allergic reactions. The drug most
likely involved is?
a. Bupivacaine
b. Lidocaine
c. Etidocaine
d. Procaine

106. These are adverse effects that may occur on administration of typical antipsychotics except?
a. Hypoprolactinemia
b. Weight gain
c. Akathisia
d. Acute dystonia

107. Extrapyramidal side effects due to the use of antipsychotics can be managed with these
drugs except?
15
a. Benzhexol
b. Amantadine
c. Bromocriptine
d. Propranolol

108. The following are clinical indications of diazepam except?


a. Anesthetic premedication
b. Status epilepticus
c. Prophylactic of seizures
d. Anxiolytic
109. An 80-year old woman with a previous history of hypothyroidism presented with a history of
abdominal pain and vomiting. She had not moved her bowels for the previous 7 days. Two weeks
earlier, her physician prescribed a combination of paracetamol and codeine to control pain in her
osteoarthritic hips. The following are likely causes of her bowel dysfunction, except?
a. Hypothyroidism
b. Paracetamol
c. Codeine
d. Aging

110. Interpersonal communication skills required in providing pharmaceutical care include all the
following except?
a. Understanding non-verbal communication
b. Physical assessment
c. Active listening
d. Use of feedback

111. The causes of inappropriate adherence include:


a. Drug product not available
b. Patient does not understand instructions
c. Patient cannot afford the drug
d. All of the above

112. Which of the following measures does not help in preventing diabetes complications?
16
a. Controlling blood glucose
b. Controlling blood pressure and blood lipids
c. Eliminating all carbohydrate from diet
d. Prompt detection of diabetic eye and kidney disease

113. Hyperinsulinemia may be caused by all of the following except?


a. Nesidioblastosis
b. An insulinoma
c. Type-1 diabetes
d. Insulin resistance
114. After a meal, blood glucose levels increase and stimulate secretion of a hormone called?
a. Glucagon
b. Glycogen
c. Pancreatisome
d. Insulin

115. Which of the following are sickle cell patients placed on for the management of anemia?
a. Iron supplements
b. Folic acid
c. Vitamin K
d. Vitamin C

116. The goals of ART include the following except?


a. Prolongation of life and the improvement of quality of life.
b. Greatest possible reduction in viral load for as long as possible.
c. Elimination of HIV entirely from the body.
d. Rational sequencing of drugs, limiting drug toxicity and facilitating adherence.

117. Which of the following is true after attaining LDL goal in dyslipidemia treatment?
a. Reduce TG levels to less than 2.0mmol/L
b. Reduce HDL to less than 1.0mmol/L
c. Increase HDL to greater than 1.0mmol/L
d. None of the options
17
118. Which of the following is contraindicated in renal insufficiency?
a. Fibrates
b. Statins
c. Niacin
d. None of the above

119. The biomarkers for coronary heart disease include:


a. Lipoprotein A
b. C-reactive protein
c. Homocysteine
d. All of the above

120. Lifestyle changes recommended is dyslipidemia include:


a. Diet
b. Exercise
c. Smoking cessation
d. All of the above

121. Drugs associated with dyslipidemia include commonly all except?


a. Calcium channel blockers
b. Thiazide diuretics
c. Oral estrogens
d. Beta blockers

122. Prognostic factors in breast cancer include:


a. Estrogen/progesterone receptor expression
b. Stage of cancer
c. Patient’s age
d. All of the above

123. Common side effects of radiation therapy include


a. Skin irritation
18
b. Swelling or breast heaviness
c. Sunburn-like skin changes
d. All of the above

124. Which of these counselling instructions is not applicable to the use of alendronate tablets?
a. Should be taken on a chosen day of the week and once weekly.
b. Should be taken first thing in the morning.
c. Patient should remain standing or walking for at least 30 minutes after the intake of the
drug.
d. Must be chewed or swallowed with plenty of water.

125. A patient with past history of peptic ulcer and chronic arthritic pain needs to be placed on
an NSAID. Which of these drugs would you recommend?
a. Naproxen
b. Celecoxib
c. Piroxicam
d. Diclofenac potassium

126. A patient on ramipril and spironolactone will most likely experience severe
a. Hypercalcemia
b. Hypernatremia
c. Hyperkalemia
d. Hypermagnesemia

127. Anemic patients with low mean corpuscular volume would benefit mainly from?
a. Ferrous gluconate
b. Folic acid
c. Hydroxocobalamin
d. Pyridoxine

128. A 5-year old accidentally ingests a brand of organophosphates used as insecticide. Which of
these drugs would you recommend to be given in addition to other supportive management?
a. Tubocurarine
19
b. Obidoxime
c. Acetylcysteine
d. Pancuronium

129. The blood M/C/S of a 2-year old boy was carried out and the drug sensitivity pattern below
was observed
Ciprofloxacin +++, Cefixime +++, Co-trimoxazole +, Azithromycin –
On the basis of this test alone, which of the underlisted drugs is the best choice?
a. Ciprofloxacin
b. Cefixime
c. Co-trimoxazole
d. Azithromycin

130. Which of the following injections is used in the management of fits in women with
eclampsia?
a. Calcium gluconate
b. Sodium bicarbonate
c. Potassium chloride
d. Magnesium sulphate

131. You have dispensed metronidazole to a patient. Which of the following instructions will you
give to the patient?
a. To take the drug before food.
b. To avoid alcohol intake while on the medication and for at least 5 days after treatment.
c. To avoid dairy products for the duration of treatment.
d. To be in supine position immediately after taking the tablet.

132. The progesterone conditions except -estrogen combination oral contraceptive is


contraindicated in the underlisted?
a. Pulmonary embolism
b. Deep vein thrombosis
c. Family history of breast cancer
d. Breast feeding women with 8-months old baby
20
133. A 35-year old woman who ingested 30 tablets of Paracetamol 500mg a few hours ago was
found unconscious and rushed to the emergency ward of the hospital. Which of the following
should be given?
a. Suxamethonium
b. N-acetylcysteine
c. Naloxone
d. Atropine

134. Which of the following is the drug of choice for treating severe malaria in a 2-month old
pregnant woman at a secondary care facility?
a. Chloroquine injection
b. Fansidar injection
c. Artemether injection
d. Quinine injection

135. Which of the following protease inhibitors is least likely to impair glucose tolerance?
a. Fosamprenavir
b. Indinavir
c. Saquinavir
d. Tipranavir

136. Judy brings in her 47-month old son Jack who has a temperature of 38oC. He has no other
symptoms. What is the most suitable dose of paracetamol 120mg/5ml suspension QDS for Jack?
a. 2.5ml
b. 5ml
c. 7.5ml
d. 10ml

137. Mrs. Emmanuel wants a word in private. She has started to feel drowsy lately and can’t
understand why. You ask about her medication history and she says that she is only on a cream
which was prescribed recently. Which of the following creams can cause drowsiness?
A. calamine
21
B. crotamiton
C. doxepin
D. tretinoin

138. Mr. SW is a 63-year old homeless man who is under your hospital’s palliative care pathway.
He has been complaining this morning of feeling nauseous and being in pain. During his stay in
hospital Mr. SW has not been eating well and is underweight. Which ONE of the following pieces
of advice is NOT correct?
a. Helping Mr. SW with his social issues is an important aspect of palliative care which should be
considered prior to discharge.
b. Mr. SW can be prescribed some dexamethasone if appropriate to help build up his appetite in
order to help him gain some weight.
c. Initiation of an opioid analgesic in Mr. SW should be delayed if there is concern over a
theoretical likelihood of addiction.
d. The cause of Mr. SW’s nausea should ideally be determined before deciding to prescribe any
anti-emetic therapy.

139. Regarding beta-adrenoceptor antagonists, which ONE of the following statements is false?
a. Carvedilol is not licensed for use in children under 18
b. Labetalol is safe to use for hypertension in pregnant women
c. Sotalol is known to prolong the cardiac QT interval
d. Atenolol has a short duration of action and can be given up to twice daily

140. From the statements below, “Prescription” means:


I. An order given individually for the person for whom prescribed, directly from the practitioner,
or the practitioner’s agent, to a pharmacist.
II. A chart order written for an inpatient specifying drugs which he or she is to take home upon
discharge.
III. A chart order written for an inpatient for use while he or she is an inpatient.

a. I only
b. I and II only
c. II and III only
22
d. All

141. Which of the following is/are considered Diabetes Risk Factor(s)?


a. Physical inactivity.
b. Women who delivered a baby greater than 9lb.
c. First-degree relative with diabetes.
d. All of the above.

142. Poison Ivy rash is an example of:


a. Type I Hypersensitivity
b. Type II Hypersensitivity
c. Type III Hypersensitivity
d. Type IV Hypersensitivity

143. Home ovulation tests usually detect a preovulatory surge in which of the following in the
urine?
a. Prolactin
b. Estrogen
c. Progesterone
d. Luteinizing hormone

144. Thiazide diuretics should NOT be used as a first line treatment for hypertension in patient
suffering from:
a. Hyperlipidemia
b. Heart failure
c. COPD
d. Gout
145. β-adrenoreceptor antagonists are not the drug of choice in the management of
hypertensive diabetic patients because they?
a. Block hypoglycemic warning signs
b. Cause glucose intolerance
c. Cause severe bradycardia
d. Increase the level of ketone bodies
23
PHARMACOECONOMICS, PHARMACY MANAGEMENT AND GENERAL MANAGEMENT
1. Process criteria in pharmaceutical care are divided into:
(a) Structure and Process (b) Technical and Interpersonal (c) Subjective and Objective (d)
Manual and Computerized.

2. Pharmacoeconomics provides evidence for health care decisions in terms of:


(a) Clinical efficacy (b) Patient Safety (c) Medication Effectiveness (d) Value for money

3. When handling a prescription in a stock out situation, which of the following actions
may the pharmacist NOT take?
(a) Inform the prescriber.
(b) If the medicine cannot be substituted with another medicine that is available, inform
the patient.
(c) If the patient agrees for it to be supplied at a later time, arrange to get stock.
(d) If the patient requires the medicine urgently, the pharmacist can substitute without
the prescriber’s permission.

4. Which of the following is NOT a barrier to community pharmacy’s involvement in the new
public health agenda?
a) A propensity for pharmacists to focus on the biomedical model of health
b) The operation of community pharmacy in a retail environment
c) A lack of cooperation between pharmacists in the community pharmacy sector
d) The location of many community pharmacies

5. Typical pharmacoeconomic studies include all of the following EXCEPT


(a) Cost-minimization analysis (b) Cost of illness (c) Cost-benefit analysis (d) Cost-utility analysis

6. The community pharmacy that can quote shares in the stock exchange market is:
(a) Private limited liability (b) Sole proprietorship (c) Public limited liability (d) Partnership

24
7. The following is not a scope of community pharmacy:
(a) Therapeutic drug monitoring (b) Point-of-care testing (c) Monitoring patients’ health (d)
Health education

8. The retailer’s selling price minus the cost price is:


(a) Mark-up (b) Margin (c) Estimated price (d) Net profit

9. Proprietary name of drug is the name given by the manufacturer as:


(a) Retailer name (b) Chemical name (c) Brand name (d) Generic name

10. The pharmacoeconomic model that considers cost in monetary units and assumes equal
outcomes is: (a) cost-benefit analysis (b) cost-utility analysis (c) cost-effectiveness analysis (d)
cost-minimization analysis.

11. While rotating stock, you notice an expiration date on a bottle that reads 08/19. What is the
last day the product may be used?
a. July 31, 2019 b. August 1, 2019 c. August 31, 2019 d. September 1, 2019
12. Which of the following steps in the planning process should be completed before the others
can be addressed?
a. Putting plan into action
b. Stating organizational objectives
c. Listing alternative ways of reaching objectives
d. Developing premises on which to base each alternative

13. The following are attributes of economic recession except


a. Period of general economic decline
b. Contraction in the GDP for 6 months (2 consecutive quarters)

c. High unemployment rate


d. Accelerated increase in wages to carter for inflation

14. Strategic management levels require a set of more of the following skill
a. Technical and human skills

25
b. Conceptual and technical skills
c. Conceptual and human skills
d. Technical skills only

15. Which of the following is the feeling of well-being that we experience in our work whether or
not we like what we do and immediate environment surrounding us and our work?
(a) Motivation b. Job enrichment c. Job satisfaction d. Employee engagement

16. Which of these are those who understand what they need to do to add value to the
organization and are satisfied enough with the organization and their roles within it to be willing
to do whatever is necessary to see it that the organization succeeds?
a. Satisfied employees

b. Motivated employees
c. Engaged employees

d. Compensated employees

17. What happens to a Public Limited Liability Company if a shareholder dies?


a. The company is dissolved
b. The company is not dissolved

c. The operations in the company will be temporarily suspended


d. The company is dissolved subject to the orders of the court

18. The reduction in the long-run and marginal cost arising from an increase in size of an
operating unit is called
a. Competitive advantage

b. Learning curve
c. Economies of scale

d. Management by exception

19. In time management, the following can be considered as important and urgent EXCEPT
a. Crises
26
b. Pressing problems
c. Relationship building
d. Deadline driven projects

20. The process of searching for and obtaining sufficient number and quality of potential job
seekers or applicant to enable the organization select the most appropriate people to fill its job
needed is called
a. Selection
b. Manpower planning

c. Recruitment
d. Training and development

21. Principal agents of the Nigerian Stock Exchange are the following
a. Merchant bankers

b. Stockbrokers and issuing houses


c. Commercial banks

d. Central bank
22. The purpose of supply chain management is:
a. Provide customer satisfaction
b. Improve quality of product

c. Integrating supply and demand management


d. Increase production

23. Knowledge, creative ideas or expressions of human mind that have commercial value and are
protectable under copyright, patent, service mark, trademark or trade secret laws from imitation,
infringement and dilution is called
a. Knowledge management
b. Learning curve
c. Intellectual property
d. Competitive advantage

24. The following is not a principle of Total Quality Management:


27
a. Management by fact

b. Respect for people


c. Top managers must take charge of all issues in the organization

d. Plan-Do-Check-Act

25. The following are true of planning except that planning


a) Reduces uncertainty
b) Helps the manager in a chaotic environment
c) Is too expensive and may erode the profit of the organization
d) Gives the opportunity for performance to be compared against established objectives

26. One of the following is not true of strategic planning process

a) Strategic formulation
b) Strategic implementation
c) Planning is short range
d) Evaluation and control

27. Increase in the degree of responsibility a worker has over a job is known as:
a. Job enlargement b. Job enrichment c. job simplification d. Job design

28. Which of the following encompasses all activities with the flow and transformation of goods
from the raw material stage, through to the end user, as well as the associated information
flows?
a. Production line b. marketing channel c. Supply chain d. Warehouse
29. Total Quality Management (TQM) emphasizes:
a. A system where strong managers are the only decision makers
b. A process where mostly statisticians get involved
c. A commitment to quality that goes beyond internal company issues to
supplies and customers
d. The responsibility of the quality control staff to identify and solve all quality-
related problems.

30. The following are true of conflict in conflict management Except:

28
a. A Periodic occurrence and fundamental part of human relationship

b. A Normal, inescapable part of life


c. An opportunity to vent an anger on the opposition

d. An opportunity to understand opposing preferences and values

31. In conflict management, one of the following is the direct expression of idea, opinions, and
desires; the intent is to communicate in an atmosphere of trust; initiate communication in a way
that conveys their concern and respect for others:
a. Passive behavior
b. Assertive behavior
c. Aggressive behavior
d. Accommodating behavior

32. Which of these bests describes e-dispensing?


a. Using the computer to electronically dispense medicines.
b. Receiving prescriptions from patients and dispensing electronically.
c. Receiving prescriptions electronically and dispensing accordingly.
d. Using a computer to completely manage the pharmacy.

33. The rate at which drugs and other goods are used, generally expressed in number of days is
called?
a. Point of Sale (POS)
b. Turnover
c. Order rate
d. Mini/Maxi stock level

34. Best practices for ordering medication include having?


a. The same person places the order and checks it in
b. Different people place the order and check it in
c. The pharmacy manager checks in all products
d. Anyone available place and receives orders

29
35. Safety stock levels are required for?
a. Selection b. Procurement c. Quantification d. Distribution

36. The anticipated unit cost for each drug multiplied by the number of units to be purchased
gives?
a. Expected purchase value for the entire quantity
b. Expected quantity to order
c. Expected purchase value for the entire inventory on hold
d. None of the options

37. Which of these business models is appropriate for registration of a community pharmacy in
Nigeria?
a. Limited Liability Company
b. Cooperative Society
c. Partnership
d. Sole proprietorship

38. Personal management involves:


a. Planning and outlining personal goals for one’s life.
b. Working towards fulfilling these goals.
c. Short-term and long-term goals that are varied.
d. All of the options.

39. Personal management skills involves the following except:


a. Managing by objectives.
b. Time management.
c. Stress management.
d. Managing difficult people.

40. The four resources that a manager must organize to run a successful organization include one
of the following:
a. Manpower, materials, motivation, and money.
b. Manpower, materials, machines, and money.
30
c. Manpower, men, materials, and money.
d. Manpower, machines, men, and motivation.

41. Critical thinking involves


a. Gathering relevant information
b. Forming an opinion
c. Making a decision
d. All of the options

42. ICT can be applied in the following areas of pharmacy practice except?
a. Dispensing
b. Identifying of drug-drug interactions
c. Monitoring patients’ adherence
d. All of the above

43. MIS stands for what and can be used for what?
a. Management Intelligence System used for surveillance.
b. Management Information System used for management decision making.
c. None of the options.
d. All of the options.

44. Which of the following statements is most correct about stress and its management?
a. Stress is a part of life; so also, is being stressed out.
b. Breathing deeply can aid in relieving stress.
c. Just saying “NO” could be a strategy for stress management.
d. Some stressors need immediate attention, while some do not.

45. Which of the following statements is true about mentoring?


a. The mentor should be older than the mentee.
b. The mentor should be more knowledgeable or more experienced than the mentee.
c. All of the options.
d. None of the options.

31
46. You are a locum working in a community pharmacy in Leeds for the first time. What should
you NOT do if a patient indicates that there has been a dispensing error?
A. ask to establish the incorrect medicine
B. tell them to come back tomorrow when the regular pharmacist is back as she made the
mistake
C. establish if the patient has taken any of the incorrect medicine
D. make a supply of the correct medicine if available

47. Advantages of pharmacy automation include the following except


a. Easy inventory management

b. Printed labels reduce compliance errors


c. Patient data cannot be retrieved

d. Shorter patient wait-time

PHARMACOLOGY AND PHARMACOTHERAPY

1. Medications with known evidence to improve survival in cardiac failure include the following
EXCEPT:
(a) ACEIs (b) Spironolactone (c) Digoxin (d) None of the options

2. All patients with a suspected acute myocardial infarction should be given aspirin to:
(a) Reduce chest pain (b) Dissolve thrombus (c) Reduce myocardial oxygen (d) All of the options

3. Which of the following may be an additional drug therapy in diabetes care?


(a) Insulin (b) Metformin (c) Lisinopril (d) Acarbose

4. Which of the following drugs may lower serum phenytoin level?


(a) Cimetidine (b) Carbamazepine (c) Metronidazole (d) Fluconazole

5. Some antihypertensive agents affect potassium levels. Which option is wrongly matched?

32
(a) Lisinopril → hyperkalemia (b) Losartan → hyperkalemia (c) Enalapril → hypokalemia (d)
Hydrochlorothiazide → hypokalemia

6. The following are cardio-selective β – adrenoreceptor antagonists EXCEPT:


(a) Timolol (b) Acebutolol (c) Metoprolol (d) Atenolol

7. A 57-year-old diabetic male is currently on long-acting insulin injection once a day. While
reviewing his medication, it was observed that postprandial glucose levels are too high. Which of
the following insulin preparations may be recommended to control his postprandial
hyperglycemia?
(a) Lente insulin (b) Insulin lispro (c) Regular insulin (d) Insulin glargine

8. Which of the following is a commonly prescribed hypnotic drug known to have anxiolytic,
muscle relaxant and anticonvulsant actions?
(a) Tramadol (b) Imipramine (c) Diazepam (d) Phenobarbitone

9. A patient is brought into the emergency room after an attempted suicide. She had consumed
three bottles of an arsenic-containing insecticide. Which of the following medications may be
administered?
(a) Dimercaprol (b) Deferoxamine (c) Amyl nitrite (d) Methylene blue

10. These contraindications apply, EXCEPT:

(a) Chloroquine – myasthenia gravis


(b) Mefloquine – seizures
(c) Chloroquine – epilepsy
(d) Ciprofloxacin – glucose-6-phosphate dehydrogenase deficiency
(e) All apply

11. Which of the following is likely to cause hyperglycemia and hypercholesterolemia in an HIV-
positive patient?
(a) Zidovudine (b) Trimethoprim-sulphamethoxazole (c) Acyclovir (d) Indinavir

33
12. Select the wrongly matched adverse effect:
(a) Chloroquine→Convulsions (b) Furosemide→ Hyperglycemia (c) Formoterol
→Bronchoconstriction (d) Codeine →Depression of respiration

13. It is of utmost importance to conduct which of the following tests before initiation of therapy
with metformin?
(a) Serum creatinine (b) Serum potassium (c) Complete blood count (d) Platelet count

14. A 7-year-old boy receiving treatment for allergic rhinitis has flushed cheeks and dilated pupils
on physical examination. These findings are due to antagonism of which receptors?
(a) Nicotinic (b) H2 (c) Muscarinic (d) H1

15. On ingestion of some tablets of chlorpheniramine, a patient experienced blurred vision. This
effect is most likely related to antagonism of which of the following neurotransmitters?
(a) Serotonin (b) Acetylcholine (c) Adrenaline (d) Histamine

16. These agents may cause hyperkalemia, EXCEPT


(a). Lisinopril (b)Hydrochlorothiazide (c) Valsartan (d) Amiloride

17. A 46-year-old nurse presents with delirium. Physical examination revealed hot and flushed
skin, dry oral mucosa, pupils poorly responsive to light and dilated. A bottle of atropine was found
in her pocket. Which of the following drugs can cause a similar clinical presentation?
(a) Prazosin (b) Amitriptyline (c) Bromazepam (d) Echothiophate

18. Which of the following drugs has the least potential to cause tolerance and addiction?
(a) Zolpidem (b) Bromazepam (c) Flurazepam (d) Estazolam

19. Which adverse effect is WRONGLY matched?


(a) Ritonavir → Hypocholesterolemia
(b) Zidovudine → Anaemia
(c) Phenytoin → Gingival hyperplasia

34
(d) Carbamazepine →Aplastic anaemia

20. A 2-year-old boy has moderate growth retardation and prominent brown stains on his teeth.
These may be attributed to the mother’s use of the following substances during pregnancy?
(a) Alcohol (b) Cocaine (c) Tetracycline (d) Aspirin

21. Which of the following substances would MOST RAPIDLY reverse the effects of warfarin?
(a) Protamine (b) Fresh frozen plasma (c) Vitamin K (d) Succimer

22. Which of the following antituberculosis drugs is known to cause high uric acid levels?
(a) Cycloserine (b) Isoniazid (c) Rifampicin (d) Pyrazinamide

23. Which drug interaction is correct?


(a) Sulphonamides significantly reduce the effects of chlorpropamide.
(b) Lamivudine antagonizes the action of nevirapine.
(c) ACEIs may decrease the plasma levels of digoxin.
(d) Co-administration of nevirapine lowers the levels of saquinavir considerably.

24. A 54-year-old male suffering from depression, is treated with sertraline. Which of these is the
most likely side effect due to sertraline?
(a) Hypotension (b) Cardiac arrhythmias (c) Sexual dysfunction (d) Weight gain

25. Adding clavulanic acid to a course of amoxicillin serves to?


(a) Delay hepatic metabolism of amoxicillin (b) Increase glomerular filtration of amoxicillin
(c) Decrease inactivation of amoxicillin by bacteria (d) Decrease renal clearance of
amoxicillin

26. A 58-year-old man taking hydrochlorothiazide for hypertension presents with muscle
weakness and cramps. Which of the following is the most likely cause of his symptoms?
(a) Hypokalemia (b) Hypoglycemia (c) Hypocalcemia (d) Hypouricemia

27. These are adverse effects of captopril, EXCEPT:


35
(a) Hypokalemia (b) Fetopathic potential (c) alteration or loss of taste (d) dry cough

28. A patient was prescribed a drug for motion sickness, which of the following adverse effects is
he likely to experience?
(a) Frequent urination (b) Diarrhea (c) Dry mouth (d) Nasal congestion

29. Which of the following may precipitate sympathetic (hypertensive) crisis?


(a) Mature cheese + bupropion (b) Tyramine + phenelzine
(c) Red wine + edrophonium (d) Metoprolol + fluoxetine

30. A 62-year-old male presents to the hospital with suprapubic pain and anuria; placement of
Foley’s catheter reveals 1000 ml of urine. He was previously diagnosed with hyperlipidemia, type
2 diabetes mellitus and hypertension. Recently, he was treated for painful peripheral neuropathy.
Which of the following drugs may have contributed to the patient’s current condition?
(a) Atorvastatin (b) Metformin (c) Tamsulosin (d) Amitriptyline

31. Common warning symptoms of a heart attack includes the following except?
(a) Chest pain or discomfort (b) Shortness of breath (c) Lightheadedness (d) Blood-shot eyes

32. On administration of IV atropine, a patient with organophosphate poisoning gradually


improves. Which of the following is still a risk for the patient?
(a) Muscle paralysis (b) Urinary incontinence (c) Bronchospasm (d) Diarrhea
33. The development of hemorrhagic cystitis in a breast cancer patient undergoing
chemotherapy may be prevented by the administration of:
(a) Ondansetron (b) Mesna (c) N-acetylcysteine (d) Folinic acid

34. A 55-year-old male treated with high-dose methotrexate experiences fever, painful mouth
ulcers and pancytopenia. Which of the following agents could have prevented his condition?
(a) Acetaminophen (b) Allopurinol (c) Ceftriaxone (d) Leucovorin

35. The following should be avoided in glucose-6-phosphate dehydrogenase deficient individuals


EXCEPT:
36
(a) Paracetamol (b) Chloroquine (c) Dapsone (d) Camphor mothballs

36. Which of the following may be used to provide anticoagulant therapy for a pregnant woman
with deep vein thrombosis?
(a) Clopidogrel (b) Aminocaproic acid (c) Warfarin (d) Heparin
37. A 53-year-old hypercholesterolemic, diabetic and hypertensive male presented to the
hospital three weeks after starting a new medication. Laboratory tests show increase in
potassium from 4.8 – 5.2 mEq/L and creatinine from 1.1 – 1.7 mg/Dl. Which of the following
drugs is most likely responsible for the clinical picture?
(a) Hydrochlorothiazide (b) Furosemide (c) Lisinopril (d) Atorvastatin

38. While at a party, a 22-year-old female taking a certain medication develops facial flushing,
headache, nausea, vomiting and abdominal cramps immediately after consuming an alcoholic
drink. Which of these drugs did she likely take?
(a) Metronidazole (b) Acetaminophen (c) Vitamin C (d) Folic acid

39. Which of the following drugs rarely causes hypoglycemia when used as monotherapy?
(a) Glibenclamide (b) Regular insulin (c) Metformin (d) Glipizide

40. In the use of medicines, the dose capable of producing marked functional derangement in
the body is called:
(a) Minimum dose (b) Maximum dose (c) Lethal dose (d) Toxic dose

41. A dose given on the initiation of therapy to give rapid drug plasma levels equivalent to that
reached after multiple dosing is called
(a) Loading Dose (b) Therapeutic dose (c) Maximum Dose (d) Maintenance dose

42. Which of the following drugs used in nausea and vomiting has the advantage of not readily
crossing the blood– brain barrier?
(a) Domperidone (b) Metoclopramide (c) Prochlorperazine (d) All of the options

43. Which of the following will make a better choice for the management of allergic rhinitis?
(a) Pseudoephedrine (b) Promethazine (c) Oxymetazoline (d) levocetirizine
37
44. Which of the following may be recommended for use in a 3-month-old baby with chronic
constipation?
(a) Glycerol suppositories (b) Bisacodyl (c) Ispaghula husk (d) Senna

45. Factors that affect drug absorption include all EXCEPT:


(a) Drug half-life (b) Gastric motility (c) Blood flow (d) Food intake

46. Which of the following is a protease inhibitor that may be used to boost the activity of
lopinavir?
(a) Stavudine (b) Indinavir (c) Ritonavir (d) Atazanavir

47. Which of the following drugs would you use with caution in patients with a history of
peripheral neuropathy?
(a) Stavudine (b) Indinavir (c) Ritonavir (d) Amprenavir

48. Which among the following is classified as an antihistamine?


(a) Chlorpromazine (b) Compazine (c) Chlorothiazide (d) Cetirizine

49. Which of the following is NOT an antiretroviral medicine?


(a) Acyclovir (b) Abacavir (c) Dolutegravir (d) Etravirine

50. Which of the following is NOT a psychotropic medicine?


(a) Sertraline (b) Fluoxetine (c) Chlorpromazine (d) Amantadine

51. One of the following medicines requires that the dose be tapered?
(a) Prednisolone (b) Nevirapine (c) Primaquine (d) Ciprofloxacin

52. A lady was prescribed 40mg furosemide once daily. What is the best time to take it?
(a) Morning (b) Afternoon (c) Evening (d) Bedtime

38
53. Which of the following medicines is recommended for use in intermittent preventive
treatment in Nigeria?
(a) Sulfadoxine/Pyrimethamine
(b) Sulfadoxine/trimethoprim
(c) Sulfamethoxazole/pyrimethamine
(d) Sulfamethoxazole/trimethoprim

54. International normalized ratio may be routinely measured once or twice a year when using
which of the following?
(a) Rivaroxaban (b) Warfarin (c) Urokinase (d) Aspirin

55. For whom among the following patients is ferrous sulphate NOT recommended?
(a) Patient on ARV medicine (b) Megaloblastic anaemic (c) Sickle cell anaemic (d) Pernicious
anaemic

56. Which of the following does NOT cause liver damage?


(a) Pyridoxine (b) Paracetamol (c) Amiodarone (d) Amoxycillin/clavulanic acid

57. What is the antidote for heparin toxicity?


(a) Protamine sulphate (b) Warfarin (c) Dopamine (d) Calcium salt
58. What is the antidote for warfarin toxicity?
(a) Coumarin (b) Heparin (c) Iron salt (d) Vitamin K

59. Nitrite laboratory test is used in which of the following?


(a) Management of angina (b) Diagnosis of urinary tract infection
(c) Diagnosis of liver disease (d) Management of ulcer

60. Which of the following immunoglobulins is responsible for the release of histamine in cells?
(a) IgE (b) IgA (c) IgG (d) IgM

61. All the following are penicillinase resistant EXCEPT:


(a) Oxacillin (b) Nafcillin (c) Flucloxacillin (d) Amoxycillin
39
62. Which of the following protocol is advisable in initiating nevirapine in a patient taking it for
the first time?
a) One dose of nevirapine is given daily for the first 14 days of treatment period
b) Two doses of nevirapine are given in two divided doses daily for the first 14 days of
treatment period
c) One dose of nevirapine is given daily for the first 28 days of treatment period
d) Two doses of nevirapine are given in two divided doses daily for the first 28 days of
treatment period

63. Which of the following drugs would you recommend for onychomycosis?
(a) Metronidazole (b) Methimazole (c) Itraconazole (d) Albendazole

64. Which of the following would prevent abortion in a pregnant woman with low luteal phase
(a) Ethinyoestradiol (b) Follicle stimulating hormone (c) Hydroxylprogestrone (d) Prolactin

65. Which the following drugs would cause kernicterus when administered in pregnant woman?
(a) Sulphamethoxazole (b) Gentamycin (c) Flucytosine (d) Lamivudine

66. Which of the following drugs improves the left ventricular ejection fraction in heart failure?
(a) Acebutolol (b) Propranolol (c) Carvedilol (d) Atenolol

67. Which of the following diluent is most appropriate for mixing ceftriaxone injection powder
to be administered intravenously?
(a) Water for injection (b) Lignocaine solution for injection
(c) Sodium bicarbonate solution for injection (d) Dextrose solution for injection

68. Adrenaline mixed with xylocaine injection will:


(a) Prolong the activity of xylocaine (b) Enhance metabolism of xylocaine
(c) Make Xylocaine soluble in the injection solvent (d) Improve the bioavailability of xylocaine

40
69. The following antifungal drug are used for both systemic and topical treatment EXECPT:
(a) Terbinafine (b) ketoconazole (c) fluconazole (d) Amphotericin B

70. After recommending an appropriate drug for motion sickness, which of the following side
effect would you warn the traveler about?
(a) Diarrhea (b) Nasal congestion (c) frequent urination (d) Dry mouth

71. Which adverse effect is matched correctly?


(a) Hemolytic anemia –methyldopa
(b) Hypouricemia – loop diuretics
(c) Increased glucose tolerance – hydrochlorothiazide
(d) Weight gain – metformin

72. Which of these drug-drug interactions is beneficial?


(a) Lopinavir + Ritonavir
(b) Hydralazine + Glucose
I Carbidopa + Pyridoxine
(d) Co-trimoxazole + Valsartan

73. During a routine checkup, the dentist observes that a 10-year-old boy with generalized tonic
seizures has overgrowth of gum tissue. The patient was most likely taking:
(a) Phenytoin (b) Diazepam (c) Ethosuximide (d) Zonisamide

74. These adverse effects are correctly matched, EXCEPT:


(a) First-dose hypotension/phenomenon – prazosin
(b) Masking of signs and symptoms of hypoglycemia – propranolol
(c) Bradycardia – salbutamol
(d) First-dose hypotension/phenomenon – losartan

75. A 67 y/o man admitted to the hospital for heart palpitation on warfarin for long term
anticoagulation to prevent atrial thrombus formation. Which of the following should be
monitored in the patient?
(a) Prothrombin time (b) Bleeding time (c) Activated partial thromboplastin
time (d) fibrinogen levels

41
76. An adult male on antituberculosis drug could not distinguish red-green color. Which of the
following is responsible?
(a) Rifampicin (b) Ethionamide (c) Ethambutol (D) Amino-salicylic acid

77. Some antihypertensive agents affect potassium levels. Which option is wrongly matched?
(a) Thiazide diuretics – Hypokalemia
(b) Enalapril- hyperkalemia
(c) Losartan- hypokalemia
(d) Hydrochlorothiazide – hypokalemia

78. A 42-year-old male treated with high-does methotrexate experiences fever, painful oral
ulceration and pancytopenia. Which of the following agents could have prevented this patient’s
condition?
(a) Ondansetron (B) Mesna (C) Allopurinol (D) Follinic Acid

79. A 57 y/o female present with worsening dyspnea on exertion and easy fatigability. Laboratory
evaluation reveals a hemoglobin level of 9.6 mg/dl. Which of the following supplements would
likely require parenteral administration in this patient?
(a) Vitamin B12 (B) Folic Acid (C) Vitamin C (D) Protein

80. The following are common adverse effects of bromazepam, EXCEPT:


(a) Anterograde Amnesia
(b) Blurred Vision
(c) Insomnia
(d) Impairment of Driving and Other Psychomotor Skills

81. These are β-lactamase inhibitors currently used clinically, EXCEPT:


(a) Tazobactam (B) Fomepizole (C) Clavulanate (D) Sulbactam

82. Which of the following is an antidote used in paracetamol overdose?


(a) Fomepizole (B) N-Acetylcysteine (C) Protamine Sulphate (D) Filgrastim

83. Which of the following drugs is not combined with artemisinin moiety for use as ACT in the
treatment of malaria in Nigeria?
(a) Mefloquine (B) Halofantrine (C) Piperaquine (D) Sulphadoxine-pyrimethamine

42
84. A diabetic patient with high level of ketone bodies would need to be immediately place on:
(a) Insulin (B) Sulphonyl urea (C) Biguanides (D) Glucagon-Like Peptide 1

85. Adding adrenaline to a lidocaine solution to be administered by subcutaneous injection can


prolong the duration of local anesthesia by:
(a) Decreased permeability of the vascular endothelium (b) precipitation of lidocaine
(c) changing the pH of the solution (d) local vasoconstriction

86. Which of the following drugs is not recommended for treatment of acne?
(a) Retinoid Cream (B) Erythromycin Cream (C) Benzoyl Peroxide Lotion (D) Clotrimazole
Cream

87. Mood stabilization in bipolar disorder is achieved with:


(a) Lithium (b) sodium valproate (c) Amitriptyline (d) carbamazepine

88. Which of these drugs is used for ripening of cervix during labor?
(a) Oxytocin (b) misoprostol (c) ergometrine (b) salbutamol

89. Which of the drugs would not benefit a patient with erosive esophagitis?
(a) Misoprostol (b) bismuth subsalicylate (c) domperidone (d) sucralfate

90. Which of the following is incorrect about intermittent preventive treatment (IPT)?
(a) Full treatment of malaria in pregnancy (b) done to prevent low birth weight (c)
done until delivery is due (d) avoided close to delivery

91. Risk of tendon damage or rupture is a well-established side effect of which of these drug
classes?
(a) Cephalosporins (b) Tetracyclines (c) Macrolides (d) Quinolones

92. The following medicines may be used for smoking cessation, EXCEPT?
(a) Varenicline (b) Leucovorin (c) Bupropion (d) Nicotine

93. Which of the following drugs is an H2-receptor antagonist?


(a) Fexofenadine (b) Loratadine (c) Ranitidine (d) Terfenadine

94. Dextromethorphan is indicated for the treatment of which of the following?


a. Nasal congestion

b. Runny nose

43
c. Productive cough

d. Dry cough

95. What is the usual dose of zidovudine?


(a) 200mg 12hrly (b) 300mg 12hrly (c) 400mg 12hrly (d) 500mg 12hrly

96. Which drug is used for the management of pneumocystis carinii in HIV patients?
(a) Levofloxacin (b) Metronidazole (c) Penicillin (d) Co-trimoxazole

97. Why is Vitamin B6 taken with isoniazid?


(a) To prevent peripheral neuropathy.
(b) To increase bioavailability of isoniazid.
(c) To prevent hepatotoxicity.
(d) To extend duration of action of isoniazid.

98. What is the first line drug for type 2 diabetes?


a. Insulin b. Metformin c. Vildagliptin d. Empagliflozin

99. Thiazide diuretics elicit their diuretic action in the?


a. Ascending loop of Henle
b. Proximal tubule
c. Descending loop of Henle
d. Distal convoluted tubules

100. Which bacterium is a common cause of peptic ulcer disease?


a. E. coli b. H. pylori c. S. aureus d. C. perfringens

101. Which of the following is a class III anti-arrhythmic drug?


a. Amiodarone b. Betaxolol c. Candesartan d. Doxazosin

102. Which of these drugs may be given to women at risk of delivering prematurely to promote
maturation of the fetus’s lungs?
a. Methotrexate b. Bortezomib c. Lansoprazole d. Dexamethasone
44
103. Which antiepileptic drug is known to cause both hair loss and curly hair?
a. Valproate b. Carbamazepine c. Phenytoin d. Ethosuximide

104. Acute attacks of gout can be prevented by?


a. Colchicine b. Allopurinol c. Uric acid d. Ascorbic acid

105. Fast-acting insulins include the following except


a. Isophane b. Lispro c. Glulisine d. Aspart

106. Which beta-blocker has the highest likelihood of causing vivid dreams / nightmares?
a. Atenolol b. Metoprolol c. Labetalol d. Propranolol

107. The following antihypertensive agents are safe in pregnancy except?


a. Labetalol b. Methyldopa c. Nifedipine d. Hydrochlorothiazide
108. The following are antihyperlipidemic drugs except?
a. HMG-CoA reductase inhibitors b. Protein synthesis inhibitors c. Fibrates d. Niacin

109. Chronic use of the following agents may induce hypertension except?
a. Cyclosporine b. Erythropoietin c. Clonidine d. Liquorice

110. Which of the following drugs does not have survival benefits in cardiac failure?
a. Metoprolol b. Timolol c. Carvedilol d. Bisoprolol

111. Which of these drug combinations for uncomplicated malaria should be taken with fatty
meals to ensure maximal bioavailability?
a. Artesunate + Amodiaquine
b. Artemether + Lumefantrine
c. Artesunate + Quinine
d. Artesunate + Sulfadoxine/Pyrimethamine

112. Whilst taking warfarin, patients who consume vitamin K-rich foods risk _______ their INR
value?
45
a. Increasing b. Decreasing c. Stabilizing d. Accelerating

113. Which of the following statements is not correct regarding the use of paracetamol?
a. Analgesic
b. Antipyretic
c. Anti-inflammatory
d. Absorption is much better on an empty stomach

114. A dry-hacking cough may be caused by


a. Carbamazepine
b. Dextromethorphan
c. Enalapril
d. Nifedipine

115. Select the correctly matched adverse effect


a. Minoxidil → hirsutism
b. Losartan → dry cough
c. Minoxidil → bradycardia
d. Prazosin → increase in LDL
116. Mechanisms of action of antihypertensive drugs include the following except?
a. Diuresis
b. Stimulation of peripheral α1 – adrenoreceptors
c. Activation of central α2 – adrenoreceptors
d. Blockade of angiotensin II receptors

117. The following are cardio-selective β-adrenoreceptor antagonists except?


a. Metoprolol b. Nadolol c. Atenolol d. Acebutolol

118. These drugs are likely to elicit impotence and loss of libido in men except?
a. Citalopram b. Enalapril c. Methyldopa d. Chlorpromazine

119. Which of these may precipitate a “cheese reaction”?


46
a. Yeast supplements + Sertraline
b. Prazosin + Fluoxetine
c. Tyramine + Phenelzine
d. Cheese + Bupropion

120. Which of the following are the first-line drugs in the treatment of Tuberculosis?
a. Streptomycin, Isoniazid, Ethambutol, Pyrazinamide
b. Amikacin, Cycloserine, Ethionamide, Para-amino salicylic acid
c. Sparfloxacin, Rifabutin, Streptomycin, Ethionamide
d. Ethambutol, Pyrazinamide, Rifampicin, Isoniazid
121. These drugs are beneficial in glaucoma except?
a. Pilocarpine b. Betaxolol c. Timolol d. Homatropine

122. The following are SERMs (Selective Estrogen Receptor Modulators) except?
a. Tamoxifen b. Clomiphene c. Ertapenem d. Ospemifene

123. Which of these enzymes is responsible for processing HIV proteins during production of new
viruses?
a. Integrase
b. Protease
c. DNA polymerase
d. Reverse transcriptase

124. Severe hematological toxicity is associated with the use of


a. Zidovudine
b. Nevirapine
c. Stavudine
d. Efavirenz

125. The following are drugs used in the treatment of gonorrhea except?
a. Ceftriaxone b. Penicillin c. Ciprofloxacin d. Metronidazole

126. Lipid-lowering medication that can be used in combination include


47
a. Statins
b. Nicotinic acid
c. Fibrates
d. All of the above

127. The atypical antipsychotic agents include all the following except?
a. Olanzapine
b. Risperidone
c. Haloperidol
d. Clozapine
128. Alpha blockers such as tamsulosin, doxazosin, and alfuzosin are used to improve symptoms
in patients with benign prostatic hyperplasia. They may also be used as add-on treatment for
which of the following conditions?
a. Persistent dry cough
b. Premature ejaculation
c. Hyperhidrosis
d. Resistant hypertension

129. Therapeutic index is a measure of?


a. Effectiveness of a drug
b. Safety of a drug
c. Blood-brain-barrier penetrating ability of a drug
d. Protein-binding capacity of a drug

130. All the underlisted drugs are valuable in the management of prostate cancer metastases
except?
a. Guserelin
b. Bicalutamide
c. Anastrozole
d. None of the above

131. Zafirlukast is:


a. A leukotriene antagonist used in the maintenance phase of asthma management.
48
b. An anti-diuretic hormone used in the management of diabetes insipidus.
c. An antibacterial agent with specific activity against Pseudomonas aeruginosa.
d. An antiplatelet aggregator used in preventing ischemic stroke.
132. Gingival hyperplasia is a side effect of which of these drugs?
a. Nystatin
b. Adenosine
c. Flurbiprofen
d. Phenytoin

133. Which of the following drugs is most likely to be safe in acute porphyria?
a. Tamsulosin
b. Tamoxifen
c. Topiramate
d. Trimethoprim

134. Which of the following statements about Folic acid is NOT TRUE?
a. It is indicated for prevention of neural tube defects in pregnancy.
b. It is indicated for the treatment of megaloblastic anemia.
c. The recommended daily allowance is 400 mg per day.
d. It is a water-soluble vitamin that has antidepressant, antiproliferative, anti-teratogenic,
gingival and anti-inflammatory effects.

135. Rolapitant is indicated for the treatment of:


a. Hypertension
b. Diabetes
c. Nausea and vomiting
d. Rheumatoid arthritis

136. Which of the following inhibits CYP 3A4?


a. Omeprazole
b. Esomeprazole
c. Pantoprazole
d. Rabeprazole
49
PHARMACEUTICAL TECHNOLOGY AND DISPENSING

1. All factors being equal, which of the following formulations has the least bioavailability:
(a) Suspension (b) Capsule (c) Emulsion (d) Solutions

2. How many milliliters of 100 units/mL soluble insulin should be administered to achieve a dose
of 20 units?
(a) 0.002 (b) 0.02 (c) 0.2 (d) 2

3. When dispensing medicines, which of the following actions may NOT be carried out?
(a) Pick the medicine by reading the label at least twice and cross-checking the medicine
name and strength against the prescription
(b) Tablets/capsules may be removed from the strip/blister when dispensing
(c) Loose capsules/tablets should be packed in a clean dry container
(d) None of the options

4. When compounding psychotropic substances, which of the options is NOT appropriate


(a) Can be carried out by any health professional
(b) Must be carried out by a licensed pharmacist
(c) Must be based on the request of a medical practitioner
(d) None of the options

5. Timolol 0.25% eye drops is equivalent to how many mg of timolol/Ml:


(a) 0.0025 mg (b) 0.025 mg (c) 0.25 mg (d) 2.5 mg

6. How much of an API (g) would be required to produce 0.5L of a 15 mg/ 10 mL solution?
(a) 7.5 g (b) 0.15 g (c) 0.75 g (d) 750 g

7. Rx Ferrous sulphate 1 g
Ascorbic acid 0.1g
50
Orange syrup 10 ml
Chloroform water to 100 ml
Fiat mistura pro inf.
In the formulation above, what is the role of chloroform water?
(a) Antioxidant (b) Acid emulsifier (c) Neutralizer (d) Vehicle

8. You are requested by a pediatrician to prepare Erythromycin ethyl succinate suspension


500mg/5mL for a patient weighing 50kg. If the dose of Erythromycin ethyl succinate is
50mg/kg/day, what volume of the suspension will you prepare for a ten-day supply?

(a) 125 mL (b) 250 mL (c) 175 mL (d) 500 mL


9. Which of the following is NOT an attribute used in standardizing pharmaceutical packages?
(a) Protection (b) Safety (c) Colour (d) Drug Delivery

10. Stress cracking is a negative feature of which of the following pharmaceutical packages?
(a) Plastics (b) Boro-silicate glass (c) Soda-lime glass (d) Rubber

11. Label for an extemporaneous product contains the following EXCEPT


(a) Full names of the excipients (b) Additional warnings (c) Appropriate expiry date (d) Name of
the product

12. Which of the following is NOT an applicable mechanism of milling in the Industry?
(a) Compression (b) Attrition (c) Reversal (d) Cutting

13. Which of the following DOES NOT principally affect the compression characteristics of
the tablets?
(a) Diluent (b) Disintegrant (c) Binder (d) Anti-adherent

14. The following methods are applicable when measuring dissolution rates EXCEPT:
(a) Beaker method (b) Flask-stirrer method (c) Flip-disc method (d) Paddle method

15. Which of the following IS NOT applicable during the preparation of a solution?

51
(a) To aid dissolution, high-viscosity liquid components should be added to those of lower
viscosity.
(b) To aid dissolution, low-viscosity liquid components should be added to those of higher
viscosity.
(c) Completely dissolve salts in a small amount of water prior to the addition of other solvent
elements.
(d) Aqueous solutions should be added to alcoholic solutions with stirring to maintain the
alcohol concentration as high as possible.

16. Which of the following dryers IS NOT applicable for drying damp solids?
(a) Tray (b) Tunnel (c) Rotary (d) Spray

17. ADME is popular abbreviation mainly associated with which of the following concepts?
(a) Pharmacodynamics (b) Pharmacogenomics (c) Pharmacoinformatics (d) Pharmacokinetics

18. Which of the following properties IS NOT important when choosing suppository bases?
(a) Melting Range (b) Solid-Fat Index (c) Hydroxyl value (d) Freezing point value

19. Changes that may occur in a poorly preserved cosmetic include all of the following EXCEPT:
(a) Emulsion products may undergo viscosity change (b) Clarity of products may be retained.
(c) Package and product may react (d) Perfumes may lose smell intensity

20. The ideal container for pharmaceutical products should have the following properties
EXCEPT:
a) Robust enough to protect the contents against crushing during handling and transport
b) Made of polymers in order to promote good patient compliance
c) Convenient to open and close
d) Made of inert materials

21. Which of the following IS NOT a mechanism for preservative action against microbial
Contamination?
(a) Destruction of cell wall material (b) Interference with microbial metabolism
52
(c) Cell wall enhancement profiling (d) Microbial protein complexation

22. Which of the following parameters IS NOT suitable for a sustained-release candidate?
(a) Greater than 75% absolute bioavailability (b) Total clearance is dose dependent (c)
Elimination half-life of 3-7 hrs (d) High apparent partition-coefficient

23. What information can be deduced from the tmax of a drug?


(a) Time a drug should be administered to get maximum response
(b) Time it takes a drug administered to reach maximum plasma concentration
(c) Time it takes for drug administered to be maximally bound to the plasma protein
(d) Time it takes for drug administered to be completely absorbed

24. Which of the following drugs will have a large volume of distribution?
a) Drugs that bind extensively to plasma protein
b) Drugs with low lipid solubility
c) Drugs that accumulate in fatty tissue
d) Drugs with larger molecule

25. Mixing is a fundamental step in most process sequences and normally carried out to:
(a) Control heat and mass transfer (b) Decrease volume of bulk powders (c) Reduce
physical and chemical reaction (d) None of the options

26. Which of the following is used in determining the volume of particles?


(a) Microscopy (b) Sedimentation (c) Sieving (d) Coulter counter

27. The mixture of solid and liquid to be filtered is known as:


(a) Filter cake (b) Filter medium (c) Filtrate (d) Slurry

28. The removal of bacteria from parenteral preparations is best accomplished by filters that
operate which of these filtration mechanisms?
(a) Impingement(b) Sieving (c) Attractive forces(d) Auto-filtration

53
29. The movement of dissolved drug through the boundary layers into the bulk solution is by
which of these?
(a) Maceration (b) Percolation (c) Diffusion (d) Leaching
30. The most widely used diluent in tablet dosage form is?
(a) Lactose (b) Tricalcium phosphate (c) Mannitol (d) Dextrose

31. Which of the following is a product of a liquid and an insoluble solid?


(a) Emulsion (b) Cream (c) Solution (d) Suspension

32. Which of the following is not an advantage of microwave drying?


(a) Solute migration is enhanced as there is uneven heating of the wet mass
(b) It provides rapid drying at fairly low temperature
(c) The bed is stationary, avoids problems of dust and attrition
(d) The equipment is highly efficient and meets GMP standards

33. All the following are factors affecting dissolution EXCEPT:


(a) Wetting (b) Particle size (c) Disaggregation (d) Vibration

34. A disintegrant is not required in one of the following tablets:


(a) Buccal tablets (b) Sublingual tablets (c) Chewable tablets (d) Double layered tablets

35. An example of biphasic liquid dosage form is:


(a) Suspension (b) Mixture (c) Syrup (d) Elixir

36. Which of the following cannot be filled into hard gelatin capsules?
(a) Capsules (b) Tablets (c) Aqueous solutions (d) Thixotropic mixtures

37. Which of the following is not an advantage of sustained release oral dosage form?
(a) Total amount of drug administered can be reduced (b) Increased reliability of therapy
(c) Administration of sustained release medication permits prompt termination of therapy
(d) The blood level oscillation characteristics of multiple dosing of conventional dosage
forms is reduced

54
38. Quality control tests of suppositories involve the following EXCEPT:

(a) Softening time (b) Breaking (c) Dissolution (d) Melting point

39. Isotonicity is required for:

(a) Multidose injections (b) Single-dose injections (c) Infusions (d) All of the options.

40. Which of these is NOT true for intramuscular route for injections?

(a) Injection is made into the muscles

(b) Oily solutions/suspensions cannot be safely administered

(c) Volume of injection does not usually exceed 5 milliliters

(d) Depot effect can be achieved

41. Microorganisms can be efficiently removed by filtration with

(a) Membrane filters (b) Ceramic filters

(c) Seitz glass filters (d) None of the option

42. All the following are merits of validation in pharmaceutical production EXCEPT:

(a) Reduce time of product recall (b) Eliminate defect costs (c)
Reduce risk of regulatory non-compliance (d) Reduce defect cost

43. Type I glass is also known as

(a) Borosilicate glass (b) Regular soda-lime glass (c) Treated soda-lime glass (d) None of the
options

44. Migration of oil phase into a packaging material observed in plastic is a good example of:

(a) Leaching (b) Sorption (c) Modification (d) Permeation

45. The filling method of a pharmaceutical liquid depends on the following except
a) Viscosity of the fluid b) Surface tension of the liquid c) volume of the bottle to be filled
d) compatibility with the materials used in the construction of the filling machine

46. The following mechanical equipment can be used for emulsification except
a) Homogenizers b) Chilsonator c) Mechanical Stirrers d) ultrasonifiers

55
47. The weight of rectal suppositories for children is
a) 1g b) 2g c) 3g d) 4 g
48. In sugar coating of tablets, sub-coating is done to

a) Prevent moisture absorption b) Round the edge and build tablet size c) smoothen the
surface d) Prevent tablet from breaking

49. During friability test, the number of revolutions required for a complete test for a batch is

a) 25 b) 50 c) 75 d) 100
50. The ability of tablet to withstand the rigors of transportation and use by patient can be
assessed by?
a) Hardness tester b) Disintegration test c) Friabilator d) Weighing balance

51. Enteric coated dosage forms release their drug content in the
a) Stomach b) jejunum c) colon d) caecum

52. The small intestine transit time is important for

a) Dosage form that release their drug quickly b) Enteric coated dosage forms which release drug in
the colon c) drugs that dissolve quickly in the intestinal fluid d) drugs not well absorbed in the
colon

53. Which of these suspension excipients could retard drug absorption


a) Gums b) sweetening agents c) flocculating agents d) all of the above

54. A liquid preparation intended for the irrigative cleansing of the vagina is called
a) Douche b) enema c) Elixir d) pessary

55. Which category of preservative is thiomersal?

a) Acidic b) azo group c) Mercurial d) Quaternary ammonium

56. Test conducted to identify the type of emulsion are the following except
a) Dilution Test b) Dye test c) conductivity d) Dissolution test

56
57. During the formulation of creams and ointment, the maximum amount of water that can be
added to a 100 g base at room temperature is called
a) Water value b) water number c) water activity d) water factor
58. The shape of vaginal suppository is

a) Oviform b) torpedo c) pencil d) caplet

59. Which of the following terms refers to urethral suppository


a) Pessary b) Aurinaria c) bougies d) Urinaria
60. Lozenges were originally named as
a) sweets b) gums c) pastilles d) none of the options

61. The main advantage of anhydrous lactose over hydrated lactose is

a) absence of Millard reaction b) impound compressibility c) impound flour d) High microbial


load

62. Which of the following is a water-soluble lubricant?

a) Stearic acid b) mineral oil c) PEG d) Magnesium Stearate

63. During tableting, the flow rate of granules from the hopper can be improved by addition of
a) Disintegrant b) lubricant c) binder d) glidant

64. In coating tablets, shellac is used as:


a) Polishing agent b) film coating agent c) Enteric coating agent d) sub-coating agent for
sugar coating

65. The amount of Zinc Oxide required to make 350% w/w of 15% Zinc Oxide paste is
a) 5.25 g b)52.5 g c)35.0 g d)3.5 g
66. In formulating a primary emulsion containing fixed oil, what is the proportion of oil: water:
emulsifier to be used
a) 3:2:1 b) 4:2:1 c) 2:2:1 d) 1: 2:1
67. Which of the following statements is correct

57
a. A creamed emulsion will reform on shaking
b. A creamed emulsion will not reform on shaking

c. A cracked emulsion will reform on shaking


d. A phase-inverted emulsion will revert to its original form on shaking
68. What specific instruction should be included on the label of all emulsions?
a. For external use only

b. Not to be taken
c. Shake the bottle before use
d. Store in a cool dry place
69. Which of the following labels would you use on this prescription (ii gtts os t.i.d prn)
a. For the eye b. take with food c. avoid alcohol d. for the ear

70. Which of the following strengths is most concentrated?


a. 0.25g in 25 mL

b. 0.2g in 50 mL
c. 0.5g in 250 mL

d. 0.002g in 2mL

71. You have received the prescription that reads (2 gtt OU BID).
What instruction will you give to the patient?
a. Two drops in both ears two times daily
b. Two drops in both eyes two times daily
c. Two drops in right ear three times daily

d. Two drops in left eye two times daily

72. Which of the following is the most effective method in reducing the risk of contamination while
making sterile preparation?
a. Wearing a class II hairnet
b. Impeccable hygiene
c. Mixing in a laminar flow hood
d. Wearing a HEPA respirator
58
73. You have been asked to divide 5 liters of pediatric suspension into an equal number of 10ml
and 15 ml unit dose dispensing cups. How many 10 ml cups will be made
a. 200 b. 325 c. 350 d. 500
74. Two medicinal agents M* and D* are to be combined in a 2:1 ratio to make a solution MD.
What quantity of each is required to make 90mL?
a. 75ml/15ml b. 15ml/75ml c. 60ml/30ml d. 30ml/60ml

75. A patient is placed on levothyroxine 0.175 mg OD. How many mcg would the patient receive
daily?
a. 0.0175 mcg
b. 1.75 mcg

c. 17.5 mcg
d. 175 mcg

76. A pharmacy stock 90% w/v solution of Drug W, you need to prepare 200 ml of a 50mg/mL
solution for a patient. What volume of the stock solution will you need?
a. 1.8 mL b. 11.1 mL c. 18 mL d. 9 mL

77. Cefaclor is available in 50mg/mL oral suspension. The doctor has written a prescription for
250mg po tid X 7 days. How would you instruct the patient’ caretaker to administer the drug?
a. Give one 5ml spoonful three times daily for seven days
b. Give one 15ml spoonful three times daily for seven days
c. Give two 5ml spoonful three times daily for seven days
d. Give one 10ml spoonful three times daily for seven days

78. How many 150mg capsules are needed to prepare 60 mL of a 1% solution


a. 3 b. 4 c. 5 d. 7

79. Insulin is available in 10 mL vials containing 100 iu/mL. Your patient is going away on a 2-week
mission in a remote village and asks how many vials they will need to take with them. If the patient
injects 42 iu bid, how many vials will be needed?
59
a. One vial b. Two vials c. Three vials d. Four vials

80. What is the use of titanium dioxide in capsule?


a) as diluent b) as opaquest c) as thickener d) as disintegrant
81. What is the time for disintegration of uncoated tablet?
a) 15 minutes b) 10 minutes c) 30 minutes d) 20 minutes

82. From the following directions, determine the number of tablets to be dispensed:

ii tabs po qqh x 5/7,


then ii tabs po tid x 5/7,
then I tab po bid x 5/7.
a. 80 tabs
b. 65 tabs
c. 110 tabs
d. 100 tabs

83. A semi-solid preparation characterized by high content of solid is called?


a. Liniment b. Cream c. Ointment d. Paste

84. Which of the following is an example of a diffusible powder?


a. Calamine BP
b. Lactose BP
c. Sodium bicarbonate BP
d. Magnesium trisilicate BP

85. Durability of a tablet to combined effects of shock and abrasion is evaluated by using?
a. Hardness tester
b. Disintegration test apparatus
c. Friabilator
d. Screw gauge

60
86. Which of the following is not added in lozenges?
a. Sweetener b. Binder c. Disintegrant d. Glidant

87. Ophthalmic solutions are sterilized by?


a. Autoclave b. Hot air oven c. Bacterial filters d. All of the options

88. Which of the following is used to adjust isotonicity?


a. NaCl b. Boric acid c. Dextrose d. All the options

89. Which of the following is not a method of determining particle size?


a. Sieving
b. Coulter current
c. Sedimentation
d. Ultrafiltration
90. Nanoparticles can be prepared in the following ways except?
a. Emulsion polymerization
b. Interfacial evaporation
c. Solvent evaporation
d. De-solvation of natural proteins

91. Drug release from a dosage form that is dependent on the amount of drug that is in the
delivery system is:
a. Zero order
b. First order
c. Second order
d. Third order

92. Compaction of granules into tablets depends on


a. Moisture content
b. Microbial content
c. Hydration capacity
d. Swelling capacity
61
93. All of the following are additives used in liquid dosage forms except?
a. Vehicles b. Oxidants c. Perfumes d. Stabilizers

94. Which of these does not contain an emulsifying agent?


a. Absorption bases
b. Hydrocarbon bases
c. Emulsifying bases
d. Water-soluble bases

95. Which of the following modified-release dosage forms is designed to release drug at a high pH?
a. Sustained-release dosage forms
b. Gastro-resistant dosage forms
c. Extended-release dosage forms
d. Gastro-retentive systems

96. Mottling occurs in tablets due to?


a. Dye migration
b. Use of too much colorant
c. Use of too little colorant
d. Wear and tear of tableting machine

97. Which of the following is not a humectant?


a. Glycerol b. Gelatin c. Shellac d. Propylene glycol

98. Which of the following parameters is not necessary in the determination of the flow of
powders?
a. Tapped density
b. True density
c. Bulk density
d. Angle of repose

99. Which of the following will not enhance the solubility of a poorly soluble substance?
62
a. Particle size reduction
b. Increased temperature
c. Increased viscosity
d. Addition of a surface-active agent

100. Which of the following statements is not correct?


a. Suspensions of ophthalmic preparations should be free from gritty particles.
b. Sedimented particles should be easily re-dispersed.
c. Particles should sediment to form a cake
d. It is essential to control the viscosity in order to enhance flow properties.

101. Which of the following is not a reason for formulating active pharmaceutical ingredients as
dosage forms?
a. For the safe and convenient delivery of accurate dosage.
b. To protect the drug substance from destructive influences of atmospheric conditions.
c. To ensure the dosage form conforms to the packaging.
d. To conceal the bitter, salty, or offensive taste or odor of a drug substance.

102. Which of the following is an excipient that is found in ‘sugar free’ preparations as a sugar
alternative?
a. Carmellose sodium
b. Compound tragacanth
c. Mannitol
d. Sucrose

103. Which of the following is an excipient that is contraindicated in neonates?


a. Carmellose sodium
b. Mannitol
c. Benzyl alcohol
d. Sucrose

63
PUBLIC HEALTH, AND INFECTIOUS DISEASES

1. Fluoridation of water is an example of a:


(a) Primary prevention strategy(b) Secondary prevention strategy
(c) Tertiary prevention strategy (d) Primordial prevention strategy

2. Knowledge sharing which encourages people’s change in lifestyle, attitudes towards


health care and health is:
(a) Public health system (b) Medical anthropology (c) Epidemiology (d) Health promotion

3. Which of the following is NOT an objective of the National Health Management Information
System (NHMIS)?
(a) To provide information to only policy-makers at the highest national level
(b) To identify major health problems
(c) To set priorities at the Local, State and National levels
(d) To assess the state of the health of the population

4. Which of the following is NOT a public health performance indicator


(a) Maternal Mortality Rate (b) Infant Mortality Rate (c) Life expectancy (d) Mean Age
5. Public health pharmacy is a component of:
a. Pharmaceutical care b. Pharmacy practice c. Health policy d. all of the options

6. Public health pharmacy is stratified into:


a. Health policy and management
b. Micro level and macro level
c. Health promotion and health education
d. All of the options
7. Which of the following correctly describes the measurement and spread of diseases in the
community
a. Morbidity is the rate of disease or proportion of diseased persons in a geographic area

64
b. An endemic is an occurrence of clearly in excess of normal expectancy

c. An endemic is a disease occurring globally and affecting a high proportion of the


population

d. Mortality is the proportion of the people who live from a disease in a geographic area
8. Herd immunity is
a. Immunity gained from person to person transaction
b. The development of immunity from previous exposure to an agent before vaccine or
natural infection

c. A direct means of immunity transfer


d. Resistance of the entire community because of people with immunity within the
community

9. Passive immunity is:


a. Immunity acquired from previous exposure to an agent before a vaccine or natural
infection

b. Resistance of the entire community because of people with immunity within the
community
c. Immunity gained from person to person transaction

d. An indirect means of immunity transfer

10. The most widely used psycho stimulant drug is?


a. Alcohol b. Nicotine c. Caffeine d. Cocaine

11. The most prevalent form of cancer in Nigeria is?


a. Breast cancer b. Prostate cancer c. Cervical cancer d. Liver cancer

12. Social determinants of health include:


a. Natural environment
b. Discrimination
c. Poor housing
d. Recreational facilities

65
13. Which of the following is an individual behavior that determines health status
a. HIV status b. Sexually transmitted diseases c. Diet d. Family history

14. Which of the following is not a scheduled immunization time?


a. 6 weeks b. 9 weeks c. 14 weeks d. 9 months
15. Which of the following pairs of vaccines are supposed to be administered at the same time?
a. HBV and Pentavalent
b. Measles and Yellow fever
c. BCG and Pentavalent

d. Meningitis and MMR and Chicken pox

16. Which of the following is not a component of the Pentavalent vaccine?


a. HBV b. Tetanus c. Diphtheria d. Rotavirus

17. Which of the following vaccine-disease pairs is not correct?


a. BCG – Tuberculosis

b. Pentavalent – whooping cough


c. Rotavirus – Gastroenteritis
d. MMR – Chicken pox – Meningitis

18. Which of the following vaccines is administered s.c?


a. DTap b. Measles c. Hepatitis B d. Rotavirus

19. Which of the following should not be included in the ADR reporting form
a. Name and demographic details of patient
b. Description and date details of the suspected ADR

c. Brand name and manufacturer details of suspected medicine


d. Name and contact detail of the reporter
e. None of the above
20. Which of the following types of health screening should not be carried out by a community
pharmacist?
a. Fasting cholesterol
66
b. Fasting blood glucose
c. Heamoglobin A1C screening

d. Antimicrobial screening

21. What is the desired healthy range (in Kg/m2) for the BMI?
a. 10.0-14.9
b. 15.0-18.5

c. 18.6-24.9
d. 25.0-29.9
22. The most common cause of maternal death is
a. Eclampsia

b. Malaria
c. Hemorrhage
d. Infection

23. Which is the most reliable way to avoid transmission of STDs


a. Consistent and correct use of male latex condoms
b. Consistent and correct use of male and female latex condoms
c. Early marriage
d. Abstinence
24. In a premarital genotype counseling, which of the following is discouraged
a. AA+AA

b. AA+AS

c. AA+SS

d. AS+AS

25. Which of the following vaccines need to be retaken after every decade
a. Influenza
b. Human papilloma virus

c. Tetanus

67
d. Hepatitis C

26. Which of the following anti-rabies immunoglobulin schedules would you recommend for a boy
bitten by a dog infected with rabies?
a. 0, 3, 7, 14, and 28 days
b. 1, 2, 7, 19, and 27 days
c. 0, 2, 9, 16, and 26 days
d. 1, 4, 11, 18, and 25 days

27. What is the causative agent for cervical cancer?


a. HIV b. HPV c. HSV d. HBV

28. What is the underlying pathophysiology of asthma?


a. Dust
b. Bronchodilation
c. Chronic air-way inflammation
d. Wheezing

29. Which of the following statements is correct?

a. Health promotion can refer to any event, process or activity that facilitates the protection
or improvement of the health status of individuals, groups, communities or populations.
b. The objective of health promotion is to prolong life and to improve quality of life.
c. Health promotion practice is often shaped by how health is conceptualized.
d. All of these

30. Obesity is caused by an increase in ______.

a. adiposity
b. epidosity
c. ediposity
d. apidosity

31. Causes of antibiotic resistance include the following except?

68
a. Under use of antibiotics
b. Over use of antibiotics
c. Use of antibiotics in farm animals
d. Antibiotic stewardship

32. Which of these is not a vaccine-preventable disease?


a. Trichomonas infection
b. Typhoid fever
c. Hepatitis A and B
d. Shingles

33. Diagnosis of gonorrhea include the following except?


a. History of sexual exposure
b. Blood test
c. Microbial examination of urethral discharge
d. Immunofluorescence using known antigen

34. Tests and procedures used to determine H. pylori infection include the following except:
a. Abdominal X-ray
b. Blood test
c. Urea test
d. Stool test
e. Scope test

35. Soil transmitted helminths include:


a. Roundworm
b. Whipworm
c. Hookworm
d. All of the options

36. Helminth infection can occur through the following except?


a. Ingestion of improperly cooked meat from infected animal.

69
b. Ingestion of cooked fish.
c. Drinking contaminated water.
d. Walking through infected soil on bare feet.

37. The BCG vaccine confers protection against which of the following diseases?
a. Brucellosis
b. Tuberculosis
c. Shingles
d. Hepatitis B
38. Miss O.Y is a 4-month old baby. Which of the following vaccines would she not have received
any dose of if her parents faithfully followed the immunization schedule?
a. Diphtheria
b. Hepatitis B
c. Measles
d. None of the options

39. One of the following is correct. Which one is?


a. Essential amino acids can be produced by the body.
b. Essential amino acids cannot be produced by the body but can be obtained from food.
c. Complete proteins are found in legumes.
d. Incomplete proteins are found in animal products.

40. Which of the following is not a method for monitoring vaccine storage?
a. Vaccine vial monitor
b. Temperature card
c. Shake test
d. None of the options

41. The worst kind of fat to consume is?


a. Linoleic acid
b. Linolenic acid
c. Omega-3 fatty acids
d. Trans fatty acids
70
42. Life expectancy is a _______
a. Morbidity indicator
b. Mortality indicator
c. Healthcare delivery indicator
d. Disability rate
43. What level of healthcare service do Federal Medical Centers belong to?
a. Primary healthcare
b. Secondary healthcare
c. Tertiary healthcare
d. None of the options

44. Gardasil is a vaccine indicated for the prevention of which of the following diseases caused by
Human Papillomavirus (HPV)?
a. Vaginal cancer
b. Hepatitis B
c. Yellow fever
d. Tetanus

45. Which of the following statements is NOT TRUE about vaccines or immunizations?
a. Two live vaccines may be given at the same time.
b. Parenteral or intranasal administration of a live vaccine is not thought to affect the immune
response of a subsequently administered oral live vaccine.
c. A live vaccine and an inactivated vaccine can be administered without regard to the timing of the
other.
d. Antipyretics/analgesics such as acetaminophen or ibuprofen should routinely be given prior to
immunization to reduce the discomfort.

46. All of the following vaccines can be stored in a refrigerator EXCEPT:


a. Influenza
b. Hep B
c. Rotavirus
d. Herpes Zoster Vaccine
71
PHARMACY LAWS, ADMINISTRATION, JURISPRUDENCE AND ETHICS.

1. The objectives of National Drug Policy (NDP) include the following, EXCEPT:

(a) To ensure efficient and effective drug management


(b) To ensure access to safe, effective, affordable and good quality drugs
(c) To enhance drug exportation
(d) To promote the rational use of drugs

2. The following are essential strategies for implementing National Drug Policy (NDP), EXCEPT:
(a) Selection of drugs (b) Procurement of drugs (c) Advertisement Policy (d) Pricing policy

3. The Essential Medicines List (EML) shall be used for the following, EXCEPT:
(a) Procurement of drugs in the public sector
(b) Prescribing drugs in the public sector
(c) Procurement of drugs in the private sector
(d) Production of Standard Treatment Guidelines

4. The following are reasons for failure of Drug Revolving Fund Scheme, EXCEPT:
(a) Poor management (b) Reimbursement of the cost of drugs for exempted patients
(c) Misapplication of the Fund (d) Purchasing of drugs at exorbitant prices

5. A body that is responsible for compiling, maintaining, and updating Hospital Formulary is
known as:
(a) Drug and Therapeutics Committee (b) Drug and Hospital Formulary Committee
(c) Hospital and Therapeutic Committee (d) Formulary and Therapeutics Committee

6. A method of detecting, assessing, preventing and managing adverse drugs reactions is called:

72
(a) Pharmacoeconomics (b) Pharmaceutical care (c) Pharmacovigilance(d) Pharmacotherapeutics

7. Systematic criteria-based appraisal of use of medicines is called:


(a) Clinical Trial (b) Drug Use Evaluation (c) Drug Use Monitoring (d) Drug Use Audit

8. Code of ethics is a set of:


(a) Fundamental legal principles (b) Rule of laws (c) Moral duties and obligations (d) All of the
options

9. Control of the counterfeit and fake drugs and unwholesome processed foods (miscellaneous
provisions) is contained in:
(a) Decree 25 of 1999 (b) Decree 25 of 1989 (c) Decree 25 of 1998 (d) Decree 25 of 2000

10. The Schedule of controlled substances is identified as


(a) Schedule I-V (b) Schedule I-X (c) Schedule 1-VI (d) Schedule1-1V

11. The National Agency for Food and Drug Administration and Control is empowered by
the following statute and as amended:
(a) Decree 14 of 1993 (b) Decree 14 of 1990
(c) Decree 15 of 1993 (d) Decree 25 of 1999

12. Pharmacy professional competency criteria include the following EXCEPT:


(a) Practicing within legal requirements (b) Demonstrating professional integrity
(c) Upholding professional standards and code of ethics (d) Registration of
Pharmaceutical products

13. Code of ethics applies to all registered Pharmacists holding licenses, certificates under the
(a) PCN Act P17 LFN 2001 (b) PCN Act P17 LFN 2002 (c) PCN Act P17 LFN 2003 (d)
PCN Act P17 LFN 2004

14. Pharmacy Jurisprudence means the science of:


(a) Laws relating to pharmacy practice but not ethics of pharmacy practice.
73
(b) Pharmacy practice but not laws of pharmacy.
(c) Ethics pertaining to pharmacy practice but not laws of pharmacy practice.
(d) Laws and ethics relating to pharmacy practice.

15. The record of the dangerous drugs must be kept for minimum of:
(a) Ten years (b) Five years (c) Four years (d) Two years
16. Drug matters are listed as:
(a) Item 19 of part 2 in the first Schedule of 1999 Constitution
(b) Item 21 of part 2 in the second Schedule of 1999 Constitution
(c) Item 21 of part 1 in the second Schedule of 1999 Constitution
(d) Item 19 of part 1 in the first Schedule of 1999 Constitution

17. The composition of the members of the Governing Council of PCN includes the following
EXCEPT:
(a) A Chairman (b) DPS of all the States of the Federation including FCT
(c) The Director-General of NAFDAC (d)The Director-General of NIPRD

18. Which of the following statements is NOT relevant to essential medicines list guiding
principle of selection?
(a) There is the need to subsidize medicines (b) Funds are limited
(c) Large numbers of drugs are available (d) Impossible to keep up-to-date with all the
drugs on the market

19. Medicines in the essential medicines list (EML) are selected with due regard to the
following EXCEPT:
(a) Public health relevance (b) Evidence on efficacy and safety
(c) Cost-effectiveness (d) NAFDAC registration

20. Which of these is NOT an advantage of generic products


(a) Names are more informative (b) Often less expensive (c) More available (d) Prescribing
facilitates substitution

21. Which of the following is NOT a function of drug and therapeutics committees?
74
(a) Develop formularies (b) Ensure adherence to treatment guidelines (c)
Approve sources of medicines (d) Engage in prescription auditing

22. Which of the following IS NOT a reason for establishing Quality Control Departments?
(a) Minimize the risk of marketing unsafe products (b) Guarantee product efficiency
(c) Guarantee patient compliance (d) Guarantee meeting regulatory requirement
23. The objectives of the drug revolving fund scheme include all except
a. Provision of good quality drugs
b. Provision of safe and affordable drugs

c. Provision of high-quality, branded drugs


d. Make drugs are available when needed

24. To sustain the drug revolving fund scheme


a. Drugs must be given freely to citizens

b. Drugs should be sold to patients


c. Drugs should be sold at replaceable cost
d. Drugs must all be imported drugs

25. All controlled drugs utilized in a public health facility should come from
a. The state government
b. The federal ministry of health
c. The federal medical stores
d. Any pharmacy store that has stock

26. Objectives of the national drug policy include all the following except
a. Make available good quality, safe and affordable drugs to Nigerians
b. Encourage local manufacturing of essential drugs
c. All of the options

d. None of the options

27. The essential drug list should be revised and reviewed every

75
a. 6 months b. 2 years c. 5 years d. 10 years
28. Advantages of UDDS do not include:
a. Patient can get drugs free of charge
b. Reduce wasting of drugs
c. Pharmacists have better access to monitor drug use by patients
d. Better quality of care for patient
29. The national health insurance scheme:
a. Enrollees pay for all the drugs needed
b. Enrollees pay 10% of value of drugs needed

c. Enrollees pay 50% of value of drugs needed


d. Enrollees pay 100% of value of drugs needed

30. Drugs included in the essential drug list shall:


a. Be listed as generic or non-proprietary names

b. Be based on the health need of the majority of the population


c. Be registered by the national drug registry authority
d. All of the options

31. The duties of drug and therapeutic committees do not include


a. The accurate estimation of the pharmaceutical requirement for the hospital
b. Procurement of pharmaceuticals for the hospitals
c. Monitoring of the use of therapeutic guidelines and overall drug utilization
d. Monitoring of the rational drug use in the hospital

32. Donated drugs to health institutions in Nigeria must have at least


a. 24 months shelf life after arrival in the country

b. 12 months shelf life after arrival in the country


c. 6 months shelf life after arrival in the country
d. 3 months shelf life after arrival in the country

76
33. Provision of drugs for only 24 hours at a time to a patient in the ward is termed
a. DOTS b. UDDS c. Clinical pharmacy practice d. Pharmaceutical care

34. Drug procurement in public health facilities should be in accordance with


a. Poison Act
b. Public procurement Act
c. Nigerian constitution

d. Pharmacy Act

35. Essential steps during procurement involve the following except


a. Appropriation
b. Needs assessment

c. Quantification
d. Counselling

36. When was the PSN established?


a)1925 b)1926 c)1927 d) 1928

37. Which of the following is not a type of PSN membership?

a. Full membership b. Associate membership c. Affiliate membership d.


Honorary membership e. None of the above

38. When was the NHIS established?


a)1984 b)1993 c)1999 d) 1997

39. Which of these bests defines pharmacovigilance?


a. Activities relating to the detection, assessment, understanding, and prevention of medicine-
related problems.

77
b. Voluntary, non-punitive, ADR incident reporting.
c. Voluntary ADR reporting for punitive measures.
d. Detection of adverse events, their sources of identification of prescribers and dispensers
responsible for them.

40. One of the following is not an aim of pharmacovigilance


a. To contribute to the assessment of benefit, effectiveness, harm, and risk of medicines.
b. To improve patient care and safety.
c. To improve public health and safety.
d. To help in identifying culprits of medication errors.

41. One of the following is not a drug quantification method


a. Consumption
b. Morbidity
c. Service-level projection
d. Market Survey

42. Which form is used to record the dispensing/sale of controlled substances?


a. Form J b. Form D c. Form K d. Form E

43. The list of medications approved for use in a healthcare system is called?
a. Formulary b. Physician’s Desk Reference
c. Encyclopedia d. Treatment Guideline

44. The National Health Insurance Scheme was established under which of the following Acts?
a. National Health Insurance Scheme Act, Cap N42 Laws of the Federation of Nigeria, 2004.
b. National Health Insurance Scheme Act, Cap N40 Laws of the Federation of Nigeria, 2005.
c. National Health Insurance Scheme Act, Cap N52 Laws of the Federation of Nigeria, 2001.
d. National Health Insurance Scheme Act, Cap N35 Laws of the Federation of Nigeria, 1999.

45. Which of the following is correct about the National Health Insurance Scheme?
a. It is a post-payment-based system.
b. It is a pre-payment-based system.
78
c. There are only two programs in the scheme: formal and informal programs.
d. It is an out-of-pocket payment scheme.

46. Drugs to be included in the Essential Drug List under the National Drug Policy should meet the
following criteria:
a. Should be listed using the generic name.
b. Must be registered with NAFDAC.
c. Should be based on the health needs of the generality of the population.
d. All of the options.

47. All of the following are to be considered in the procurement of drugs under the NDP except?
a. Procurements in the public sector shall be based only on generic names.
b. Procurements shall be limited to only drugs registered in Nigeria and are in the EDL.
c. Preference should be given to high-quality imported drugs on EDL with NAFDAC registration.
d. Procurement should be based on accurate quantification.

48. Which of the following is an impediment to the successful implementation of the DRF in
Nigeria?
a. Funds misapplication
b. Purchase of drugs at exorbitant prices
c. Delay in payment of suppliers
d. All of the options

49. Which of the following forms is filled in application for preregistration of pharmacists?
a. Form C
b. Form J
c. Form D
d. Form F

50. Disposal of expired medications at the pharmacy store should be carried out by?
a. NDLEA
b. NAFDAC
79
c. PCN
d. Superintendent Pharmacist at the pharmacy.

51. Ethical consideration requires:


a. Continuity of care in the event of labour disputes, pharmacy closure or conflict with personal
moral beliefs.
b. Serving the needs of the individual, the community, and the society.
c. Cooperation with colleagues and other professionals.
d. All of the options.

52. “A pharmacist has the duty to uphold the law, uphold the dignity and honor of the profession
and accept its ethical principles”. This statement
a. Is sometimes true
b. Is always true
c. Is never true
d. Depends on the patient

53. The objectives of the Pharmaceutical Society of Nigeria include all the following except to
a. Maintain a high standard of professional ethics and discipline.
b. Ensure branches of the society exists in all states of the Federation.
c. Establish and maintain a high standard of pharmaceutical education in Nigeria.
d. Influence legislation for the enhancement of the image of the Pharmacy profession and its
practitioners.
54. Which of the following is responsible for the day to day control of activities in a registered
pharmaceutical retail premise?
a. Pharmacist Director
b. General Manager
c. The Director of pharmaceutical services
d. Superintendent pharmacist

55. According to the “Counterfeit and Fake Drugs and Unwholesome Processed Foods” decree, a
fake drug means all of the following except?
a. Any drug or product which is not what it purports to be.
80
b. Any drug or product which is so colored, coated, powdered, or polished that the damage is
concealed or which is made to appear to be better or of greater therapeutic value than really is.
c. Any drug which is not registered with NAFDAC.
d. None of the options.

56. The Chairman of the council holds office for?


a. A period of three years only.
b. A period of six years uninterrupted.
c. A period of three years and a maximum of six years.
d. As the council deems fit.

57. Intra-professional conflict refers to conflict between members of the same profession. This
could be caused by these factors, except?
a. Increased group size
b. Increased diversity (i.e. gender, age, race)
c. Lack of information sharing
d. None of the options

58. Penalties for unprofessional conduct arise if a registered person:


a. Is convicted by any Court or Tribunal in Nigeria or elsewhere, of an offence incompatible with
the status of a pharmacist.
b. Is guilty of infamous conduct in any professional respect.
c. Has his/her name fraudulently registered with PCN.
d. All of the options

59. NDLEA was established to enforce laws against?


a. Cultivation, processing, and sale of hard drugs.
b. Trafficking and use of hard drugs.
c. Investigating persons suspected to have dealings in hard drugs.
d. All of the options.
60. The following are technical groups in the PSN except?
a. National Association of Pharmacists in Academia.
b. Association of Community Pharmacists in Nigeria.
81
c. Association of Lady Pharmacists.
d. National Association of Hospital, and Industrial Pharmacists.

61. The following are council members of PSN except?


a. All National Executive members, immediate past presidents, and all states branch chairmen.
b. Director of Pharmaceutical Services, DGs of NAFDAC and NIPRD, and the registrar of PCN.
c. Heads of Departments in the Faculties of Pharmacy in Nigeria.
d. Representatives of Armed Forces, Customs, and Nigerian Police Force who must be registered
pharmacists.

62. The PCN disciplinary tribunal consists of


a. Chairman of the council
b. Six other members appointed by the council
c. The registrar
d. The chairman and six other members appointed by the council

63. Which of the following statements is not true about Pharmacy and Therapeutics Committees?
a. It is composed only of pharmacists.
b. It oversees all aspects of drug therapy in a hospital.
c. May be involved in drug utilization reviews.
d. None of the options.

64. Which of the following is not a principle of medicines optimization?


a. Develop your professional knowledge and competence.
b. Ensure medicines use is as safe as possible.
c. Evidence-based choice of medicines.
d. Prioritize selling only the most expensive medicines.

PHARMACOGNOSY

1. The following are benefits of integration of traditional and orthodox medicines EXCEPT:
82
(a) it will improve the quality and value of research in traditional medicine
(b) it guarantees less access to healthcare delivery for people in the rural areas
(c) if traditional medicine use is covered by insurance, patronage will increase
(d) it will promote the proper use and development of traditional medicine practice

2. Choose the INCORRECT statement:


a) Traditional medicine relies on past experience
b) Traditional medicine practices are always explicable
c) Traditional medicine is used in diagnosis and prevention of diseases
d) Traditional medicine is a combination of knowledge and practices

3. Spinal cord injury is BEST managed by one of the following alternative medical practices:
(a) Acupuncture (b) Chiropractic
(c) Naturopathy (d) Ayurveda

4. Materia medica contains herbal remedies of:


(a) Indian origin (b) Chinese origin
(c) Hippocrates (d) Unani

5. For identification of a medicinal plant, the following parts are required EXCEPT:
(a) Root (b) Leaf (c) Flower (d) Seed

6. Macroscopic evaluation of a medicinal plant includes the following characteristics EXCEPT:


(a) Size (b) Shape (c) Histology (d) Texture

7. Which of the following is a physical, mental and spiritual practice that aims to transform body
and mind?
(a) Ayurveda (b) Unani (c) Homeopathy (d)Yoga

8. Which of the following is complementary medical therapy?


(a) Acupuncture (b) Traditional Chinese Medicine (c) Ayurveda (d) Naturopathy

83
9.Choose the INCORRECT statement:
(a) Garlic capsules combined with antidiabetic medications can cause an increase in blood sugar.
(b) Orange juice decreases the absorption of aluminum in antacids containing aluminum.
(c) Orange juice should be avoided when taking antibiotics
(d) Grapefruit juice interacts with calcium channel blockers

10. A herbal preparation made by placing plant materials in cold water, bringing it to boil,
simmering for a few minutes before decanting is:
(a) Concoction (b) Decoction (c) Extraction (d) Infusion

11. Tinctures are prepared by soaking herb in?


a. Alcohol b. Water c. Oil d. Glycerol

VETERINARY PHARMACY
1. Which of the following devices is not used for administering topical medicaments to
ruminants?
a. Dips
b. Pour-ons
c. Drenching guns
d. Dust bags

2. Which of the following collars is used in preventing animals from licking topically administered
medicaments?
a. Vaporous collar
b. Fluidized collar
c. Elizabethan collar
d. Skinoment collar

3. Poultry birds are not vaccinated against which of the following diseases?
(a) Newcastle (b) Bronchitis (c) Encephalomyelitis (d) Intestinal worms

84
85

You might also like